You are on page 1of 62

Total Marks : 200

Online Prelims TEST - 20 (SUBJECT WISE) Mark Scored : 0


( InsightsIAS Mock Test Series for UPSC Preliminary Exam 2020 )

1 Consider the following statements regarding Common Risk Mitigation Mechanism (CRMM)
1. It is under the Paris Agreement
2. The objective of CRMM is de-risking and reducing the financial cost of renewable energy projects.

Which of the statements given above is/are correct?


A. 1 only
B. 2 only
C. Both 1 and 2
D. Neither 1 nor 2

Correct Answer : B

Answer Justification :

m
Common Risk Mitigation Mechanism is under the International Solar Alliance. It has been taken

o
over by the World Bank and it has mobilised $1 million for preparatory work. Hence, statement 1

. c
is incorrect.

u b
The objective of CRMM is de-risking and reducing the financial cost of renewable energy projects in
ISA member countries. It will act as a pooled insurance with limited liability. Hence, statement 2

h
is correct.

te e
2 Consider the following statements regarding Petcoke

u
1. Coal is much more potent pollutant than Petcoke.

.t
2. It is a key input material for cement producers.

w
Which of the statements given above is/are correct?

w
A. 1 only
B. 2 only

w
C. Both 1 and 2
D. Neither 1 nor 2

Correct Answer : B

Answer Justification :

Petcoke is much more potent pollutant than coal and causes greater harm to the
environment and health. Normal coal attracts a clean-energy levy that has risen to Rs.400 a
metric ton. On the other hand, petcoke has been exempt from this levy. Hence, statement 1 is
incorrect.

Petcoke is the coke that, in particular, derives from a final cracking process—a thermo-based
chemical engineering process that splits long chain hydrocarbons of petroleum into shorter
chains—that takes place in units termed coker units. It is a key input material for cement producers.
Hence, statement 2 is correct.

1
Total Marks : 200
Online Prelims TEST - 20 (SUBJECT WISE)
( InsightsIAS Mock Test Series for UPSC Preliminary Exam 2020 ) Mark Scored : 0

3 Acid rain is caused by which of the following emissions?

A. Nitrous oxide and carbon monoxide


B. Sulfur dioxide and Nitrogen gas
C. Sulfur dioxide and Nitrogen oxide
D. Ammonium hydroxide and Sulfuric acid

Correct Answer : C

Answer Justification :

Acid rain is caused by emissions of sulfur dioxide and nitrogen oxide, which react with the water
molecules in the atmosphere to produce acids. Acid rain has been shown to have adverse impacts

m
on forests, freshwaters and soils, killing insect and aquatic life-forms, causing paint to peel,
corrosion of steel structures such as bridges, and weathering of stone buildings and statues as well

o
as having impacts on human health. Hence, option C is correct.

4 Consider the following statements regarding Ecotone

b . c
u
1. Ecotone often have a large number of species and larger population densities than the communities

h
on either side.

e
2. Ecotone doesn’t present in temperate region

te
3. Ecotone may be very narrow or quite wide.

u
Which of the statements given above is/are correct?

.t
A. 2 only
B. 1 and 3 only

w
C. 2 and 3 only

w
D. 1, 2 and 3

w
Correct Answer : B

Answer Justification :

An Ecotone is a transitional area of different ecosystems, such as forest and grasslands. An ecotone
may exist along a broad belt or in a small pocket, such as forest clearing, whereas two local
communities blend together.

Ecotone often have a large number of species and larger population densities than the communities
on either side. Hence, statement 1 is correct.

Ecotone does present in almost all the parts of the world. Hence, statement 2 is incorrect.

Ecotone may be very narrow or quite wide. It has conditions intermediate to the adjacent
ecosystems. Hence it’s zone of tension. Hence, statement 3 is correct.

2
Total Marks : 200
Online Prelims TEST - 20 (SUBJECT WISE)
( InsightsIAS Mock Test Series for UPSC Preliminary Exam 2020 ) Mark Scored : 0

5 Consider the following statements regarding Desert ecosystem


1. Desert plants show phenomenon of Allelopathy.
2. Desert plants possess waxy leave to arrest transpiration

Which of the statements given above is/are correct?


A. 1 only
B. 2 only
C. Both 1 and 2
D. Neither 1 nor 2

Correct Answer : C

Answer Justification :

m
Desert plants show phenomenon of Allelopathy i,e, they secrete some chemical substance which in

o
habits the growth of plants growing in their near vicinity. Hence, statement 1 is correct.

b . c
Desert plants possess waxy leave to arrest transpiration Hence, statement 2 is correct.

6 Consider the following statements regarding wetlands

h u
e
1. A mire is a wetland characterized by thick forest cover.

te
2. The marsh wetlands are dominated by herbaceous rather than the woody plants.

u
Which of the statements given above is/are correct?

.t
A. 1 only
B. 2 only

w
C. Both 1 and 2

w
D. Neither 1 nor 2

w
Correct Answer : B

Answer Justification :

Wetlands are defined as lands transitional between terrestrial and aquatic eco-system where the
water table is usually at or near the surface or the land is covered by shallow water.

Main types of wetland

Marsh: it is a wetland that is dominated by herbaceous rather than woody plant species.
These can often be found at the edge of lakes and streams. Where they form a transition
between the aquatic and terrestrial ecosystems. Hence, statement 2 is correct.

Swamp: a swamp is a wetland that is forested. Swamps are characterized by slow-moving to


stagnant water. They are usually associated with adjacent rivers or lakes. The water of swamp

3
Total Marks : 200
Online Prelims TEST - 20 (SUBJECT WISE)
( InsightsIAS Mock Test Series for UPSC Preliminary Exam 2020 ) Mark Scored : 0

may be fresh water, brackish water or seawater.

Mire: a mire is a wetland without forest cover, dominated by peat-forming plants. There are
two types of mires i,e,. BOG and FEN. Hence, statement 1 is incorrect.

7 Which of the followings are major causes for Water Pollution


1. Urbanization
2. Industries
3. Agriculture
4. Religious and Social Practices
5. Withdrawal of water and drying up of water bodies

Which of the statements given above is/are correct?

m
A. 1, 2, 4 and 5 only

o
B. 1, 2, 3 and 4 only

c
C. 1, 3, 4 and 5 only

.
D. 1, 2, 3, 4 and 5

u b
h
Correct Answer : D

Answer Justification :

te e
Water pollution is the contamination of water bodies (e.g. lakes, rivers, oceans, aquifers, and

tu
groundwater). Water pollution occurs when pollutants are directly or indirectly discharged into

.
water bodies without adequate treatment to remove harmful compounds. Water pollution affects

w
plants and organisms living in these bodies of water. In almost all cases the effect is damaging not
only to individual species and populations but also to the natural biological communities. Humans

w
aren’t the only ones facing issues related to water pollution. There are many terrestrial animals that

w
have faced extinction due to water shortage and contamination and the extinction rate of aquatic
animals is five times worse than those of land animals.

The major causes of water pollution in India are. Hence, all the above statements are correct.

Urbanization
Industries
Agriculture
Religious and Social Practices
Withdrawal of water and drying up of water bodies.

8 Consider the following statements regarding Black Carbon


1. Black carbon is the result of incomplete combustion of fossil fuels, biofuel, and biomass.
2. It is directly emitted in the atmosphere as fine particles
3. Life time of black carbon in the atmosphere is more than 100 years.

4
Total Marks : 200
Online Prelims TEST - 20 (SUBJECT WISE)
( InsightsIAS Mock Test Series for UPSC Preliminary Exam 2020 ) Mark Scored : 0

Which of the statements given above is/are correct?


A. 2 only
B. 1 and 2 only
C. 2 and 3 only
D. 1, 2 and 3

Correct Answer : B

Answer Justification :

Black carbon (BC) is the result of incomplete combustion of fossil fuels, biofuel, and biomass. It
consists of elemental carbon in several forms. Black carbon warms the atmosphere due to its
absorption and by reducing albedo when deposited on snow and ice. Hence, statement 1 is
correct.

o m
It is directly emitted in the atmosphere as fine particles PM5. It is highly effective in absorbing solar

. c
energy and can absorb million times more energy than Carbon Dioxide. Hence, statement 2 is
correct.

u b
Life time of black carbon in the atmosphere is only a few days to weeks, compared to CO2 which

h
has an atmospheric lifetime of more than 100 years. Hence, statement 3 is incorrect.

te e
9 Consider the following statements regarding Waste Minimization Circles (WMC)

u
1. It helps small and medium industrial clusters in waste minimization in their industrial plants.

t
2. It is funded by the World Bank

w .
Which of the statements given above is/are correct?
A. 1 only

w
B. 2 only

w
C. Both 1 and 2
D. Neither 1 nor 2

Correct Answer : A

Answer Justification :

Waste Minimization Circles (WMC) helps Small and Medium Industrial Clusters in waste
minimization in their industrial plants. Hence, statement 1 is correct.

This is sponsored by Ministry of Environment and Forests acting as the nodal ministry. The
project is being implemented with the assistance of National Productivity Council (NPC), New Delhi.
The initiative also aims to realize the objectives of the Policy Statement for Abatement of Pollution
(1992), which states that the government should educate citizens about environmental risks, the
economic and health dangers of resource degradation and the real economic cost of natural

5
Total Marks : 200
Online Prelims TEST - 20 (SUBJECT WISE)
( InsightsIAS Mock Test Series for UPSC Preliminary Exam 2020 ) Mark Scored : 0

resources. Hence, statement 2 is incorrect.

10 Miyawaki afforestation, sometime seen in the news, is related to

A. Afforestation of degraded lands near mining areas


B. Urban afforestation by turning backyards into mini forests.
C. Method of afforesting highly degraded lands in tropics
D. Traditional method of raising forest in North Eastern India

Correct Answer : B

Answer Justification :

m
The Miyawaki method of afforestation revolutionized the concept of urban afforestation by turning

o
backyards into miniforests.

. c
Kerala is to take this method of afforestation to add green cover on government office premises,
residential complexes, school premises etc in Kerala.

b
aki-forests/article30479463.ece

h u
https://www.thehindu.com/news/cities/Thiruvananthapuram/kerala-to-take-more-cover-under-miyaw

te e
11 Consider the following statements regarding India State of Forest Report 2019

u
1. The total tree and forest cover in the country increased by 50000 square kilometer in the last two

.t
years
2. Madhya Pradesh, Arunachal Pradesh and Maharashtra are the states having largest forest cover in

w
the country

w
3. The loss of forests was one of the causes behind rainfall deficit in the North East this monsoon.

w
Which of the statements given above is/are correct?
A. 1 only
B. 3 only
C. 1 and 3 only
D. 1, 2 and 3

Correct Answer : B

Answer Justification :

The total tree and forest cover in the country increased by 5,188 square kilometer in the last two
years. Hence Statement 1 is incorrect.

There is an increase of 42.6 million tonnes in the carbon stock of the country as compared to
the last assessment of 2017.

6
Total Marks : 200
Online Prelims TEST - 20 (SUBJECT WISE)
( InsightsIAS Mock Test Series for UPSC Preliminary Exam 2020 ) Mark Scored : 0

India has set a target of bringing 33 per cent of its geographical area under forest cover.

The TFC (total forest cover) of India in 2019 is 21.67 per cent of the total geographical area
(TGA) of the country as against 21.54 per cent (of TGA) in 2017.

The forest cover of six states, excluding Assam, has decreased by nearly 18 per cent
between 2011 and 2019.

Madhya Pradesh, Arunachal Pradesh and Chattisgarh are the states having largest forest
cover in the country (Area wise). Hence Statement 2 is incorrect.

The loss of forests was one of the causes behind rainfall deficit in the North East this

m
monsoon. Hence Statement 3 is correct.

12 Consider the following statements

. co
b
1. Flamingo Festival is held every year to promote tourism in Chilka Lake.

u
2. Nellapattu Bird Sanctuary is located in Kerala.

e
Which of the statements given above is/are correct?

h
te
A. 1 only
B. 2 only

u
C. Both 1 and 2

t
D. Neither 1 nor 2

Correct Answer : D

w .
w w
Answer Justification :

Both the statements are incorrect.

Flamingo Festival

It is held every year to promote tourism in Pulicat and Nellapattu. Flamingo Festival is being
organized for the past 12 years.

Nellapattu Bird Sanctuary is one of the biggest habitats for some hundreds of pelicans and other
birds. It is located about 20 km north of the Pulicat Lake, Nellore district, Andhra Pradesh.

13 Consider the following statements regarding Indian Renewable Energy Development Agency
1. It is a Mini Ratna Government of India Enterprise under the administrative control of Ministry of
New and Renewable Energy (MNRE).
2. It provides financial support to specific projects and schemes for generating electricity and / or
energy through new and renewable sources and conserving energy through energy efficiency.

7
Total Marks : 200
Online Prelims TEST - 20 (SUBJECT WISE)
( InsightsIAS Mock Test Series for UPSC Preliminary Exam 2020 ) Mark Scored : 0

Which of the statements given above is/are correct?


A. 1 only
B. 2 only
C. Both 1 and 2
D. Neither 1 nor 2

Correct Answer : C

Answer Justification :

Indian Renewable Energy Development Agency Limited (IREDA) is a Mini Ratna (Category – I)
Government of India Enterprise under the administrative control of Ministry of New and Renewable
Energy (MNRE). IREDA is a Public Limited Government Company established as a Non-Banking
Financial Institution in 1987 engaged in promoting, developing and extending financial assistance

m
for setting up projects relating to new and renewable sources of energy and energy

o
efficiency/conservation with the motto: “ENERGY FOR EVER”

Functions:

b . c
h u
1. To give financial support to specific projects and schemes for generating electricity and / or

e
energy through new and renewable sources and conserving energy through energy efficiency.

te
2. To strive to be competitive institution through customer satisfaction.

.tu
3. To maintain its position as a leading organization to provide efficient and effective financing

w
in renewable energy and energy efficiency / conservation projects.

w w
14 Consider the following statements regarding National Afforestation Programme (NAP)
1. Objective of the scheme is ecological restoration of degraded forests and to develop the forest
resources with peoples’ participation.
2. NAP is a centrally sponsored scheme fully funded by Central government.

Which of the statements given above is/are correct?


A. 1 only
B. 2 only
C. Both 1 and 2
D. Neither 1 nor 2

Correct Answer : A

Answer Justification :

National Afforestation Programme

8
Total Marks : 200
Online Prelims TEST - 20 (SUBJECT WISE)
( InsightsIAS Mock Test Series for UPSC Preliminary Exam 2020 ) Mark Scored : 0

The overall objective of the National Afforestation Programme (NAP) scheme is ecological
restoration of degraded forests and to develop the forest resources with peoples’ participation.

The scheme is implemented by three tier institutional setup through the State Forest Development
Agency (SFDA) at the state level, Forest Development Agency (FDA) at the forest division level and
Joint Forest Management Committee (JFMCs) at village level.

The major components of the scheme includes afforestation under Seven plantation models,
maintenance of previous years plantations and Ancillary Activities like soil and moisture
conservation activities (SMC), fencing, overheads, micro-planning, awareness raising, Entry Point
Activities (EPA) etc.

NAP is a centrally sponsored scheme which is implemented with the fund sharing pattern
of 60: 40 percent between Centre and States wherein the sharing pattern for Northeastern and
hilly States is 90:10. Hence Statement 2 is incorrect.

15 Global Climate Risk Index, sometime seen in the news, is published by

o m
Which of the statements given above is/are correct?

b . c
u
A. IUCN
B. WWF

h
C. UNEP

e
D. None of the above

Correct Answer : D

tu te
w
Answer Justification :
.
w
Global Climate Risk Index is annually published by environmental think tank Germanwatch.

w
Germanwatch, based in Bonn and Berlin (Germany), is an independent development and
environmental organisation which works for sustainable global development.

16 Consider the following statements regarding Tiger corridor


1. It is a stretch of land linking tiger habitats, allowing movement of tigers, prey and other wildlife.
2. It is not protected under any law of the land.

Which of the statements given above is/are correct?


A. 1 only
B. 2 only
C. Both 1 and 2
D. Neither 1 nor 2

Correct Answer : A

9
Total Marks : 200
Online Prelims TEST - 20 (SUBJECT WISE)
( InsightsIAS Mock Test Series for UPSC Preliminary Exam 2020 ) Mark Scored : 0

Answer Justification :

A tiger corridor is a stretch of land linking tiger habitats, allowing movement of tigers, prey and
other wildlife.

Without corridors tiger habitat can become fragmented and tiger populations isolated leaving
the tigers vulnerable to localised extinction.

The corridors are operated under Tiger Conservation Plan.

The plan is protected and implemented under section 38V of the Wildlife
(Protection) act, 1972. Hence Statement 2 is incorrect.

17 Consider the following statements regarding Sumatran rhinos

o m
c
1. Sumatran rhinos are the smallest of the living rhinoceroses and the only Asian rhino with two

.
horns.

b
2. The Sumatran rhino once roamed as far away as the foothills of the Eastern Himalayas in Bhutan

u
and eastern India

h
3. The species only survives on the Indonesian islands of Sumatra and Borneo.

te e
Which of the statements given above is/are correct?
A. 1 and 2 only

u
B. 2 only

.t
C. 2 and 3 only
D. 1, 2 and 3

w
Correct Answer : D
w
w
Answer Justification :

All the statements given above are correct.

Sumatran rhinos are the smallest of the living rhinoceroses and the only Asian rhino with two horns.

The Sumatran rhino once roamed as far away as the foothills of the Eastern Himalayas in Bhutan
and eastern India, through Myanmar, Thailand, possibly to Vietnam and China, and south through
the Malay Peninsula.

Today, the species only survives on the Indonesian islands of Sumatra and Borneo. Experts believe
the third subspecies is probably extinct.

https://www.worldwildlife.org/species/sumatran-rhino

18 Consider the following statements regarding Olive Ridley Turtles

10
Total Marks : 200
Online Prelims TEST - 20 (SUBJECT WISE)
( InsightsIAS Mock Test Series for UPSC Preliminary Exam 2020 ) Mark Scored : 0

1. It is critically endangered species


2. It inhabits warm waters of the Pacific, Atlantic and Indian oceans.

Which of the statements given above is/are correct?


A. 1 only
B. 2 only
C. Both 1 and 2
D. Neither 1 nor 2

Correct Answer : B

Answer Justification :

The Olive ridley turtles are the smallest and most abundant of all sea turtles found in the world. It

m
inhabits warm waters of the Pacific, Atlantic and Indian oceans. Hence Statement 2 is correct.

Conservation Status:

. co
u b
The Olive ridley turtle is recognized as Vulnerable by the IUCN Red list. Hence

h
Statement 1 is incorrect.

te e
International trade in these turtles and their products is banned under CITES Appendix I.

.tu
19 Colombo Declaration, sometime seen in the news, is related to

w
A. Sustainable Development

w
B. Sustainable Nitrogen Management.
C. Mitigation of Climate Change

w
D. None of the above

Correct Answer : B

Answer Justification :

Colombo Declaration on Sustainable Nitrogen Management

The aim of the Declaration is to halve nitrogen waste by 2030.

It calls upon UN agencies, other international organizations, development partners,


philanthropic agencies, academic and civil society organizations to support its
implementation.

11
Total Marks : 200
Online Prelims TEST - 20 (SUBJECT WISE)
( InsightsIAS Mock Test Series for UPSC Preliminary Exam 2020 ) Mark Scored : 0

It urges countries to conduct a comprehensive assessment on nitrogen cycling covering


policy, implementation, regulation, and scientific aspects at a national level plus sensitize the
citizens to understand the natural nitrogen cycle and how human impacts alter its balance.

The Declaration was developed with technical support from International Nitrogen
Management System (INMS).

20 Consider the following statements


1. Forest Advisory Committee is a statutory body
2. Wildlife protection Act, 1972 defines the procedure to be followed for declaring an area to be a
Reserved Forest

Which of the statements given above is/are correct?

m
A. 1 only

o
B. 2 only

c
C. Both 1 and 2

.
D. Neither 1 nor 2

u b
h
Correct Answer : A

Answer Justification :

te e
Forest Advisory Committee (FAC) of Ministry of Environment, Forests & Climate Change

tu
(MoEF&CC) has been reconstituted vide order dated 10 October 2016 (see attached) under the

.
Forest (Conservation) Act 1980. Hence statement 1 is correct

w
Indian forest act 1927

w w
It was largely based on previous Indian Forest Acts implemented under the British. (Indian
Forest Act of 1878).

Both acts sought to consolidate and reserve the areas having forest cover, or significant
wildlife, to regulate movement and transit of forest produce, and duty leviable on timber and
other forest produce.

It also defines the procedure to be followed for declaring an area to be a Reserved


Forest, a Protected Forest or a Village Forest. Hence Statement 2 is incorrect.

It defines:

what is a forest offence,

12
Total Marks : 200
Online Prelims TEST - 20 (SUBJECT WISE)
( InsightsIAS Mock Test Series for UPSC Preliminary Exam 2020 ) Mark Scored : 0

what are the acts prohibited inside a Reserved Forest

Penalties leviable on violation of the provisions of the Act

21 Consider the following statements


1. Critical wildlife habitat is defined only in the Scheduled Tribes and Other Traditional Forest
Dwellers (Recognition of Forest Rights) Act, 2006
2. Section 38V of the Wildlife (Protection) Act, 1972 explains the core or critical tiger habitat as well
as the buffer or peripheral area of a tiger reserve.

m
Which of the statements given above is/are correct?

o
A. 1 only

c
B. 2 only

.
C. Both 1 and 2

b
D. Neither 1 nor 2

Correct Answer : C

h u
Answer Justification :

te e
tu
Both the statements given above are correct.

w .
The Minister of State for Environment and Forests clarified the issues regarding Scheduled Tribes
and Other Traditional Forest Dwellers (Recognition of Forest Rights) Act in tiger reserves and in the

w
critical wildlife habitats. The Minister states that:

w
1. “Section 38V of the Wildlife (Protection) Act, 1972 (as amended in 2006) explains the core or
critical tiger habitat as well as the buffer or peripheral area of a tiger reserve.

2. A tiger reserve includes two parts:

A. Core or critical tiger habitat (National Park or Sanctuary status).

B. Buffer or peripheral area.

3. The phrase ‘core or critical tiger habitat’ is mentioned only in the Wildlife (Protection) Act, 1972,
as a sequel to amendment made to the said Act in 2006. It is NOT defined in the Scheduled Tribes
and Other Traditional Forest Dwellers (Recognition of Forest Rights) Act, 2006.

4. The phrase ‘critical wildlife habitat’ is defined only in the Scheduled Tribes and Other Traditional
Forest Dwellers (Recognition of Forest Rights) Act, 2006, and NOT in the Wildlife (Protection) Act,
1972.

5. ‘Core or critical tiger habitat’ is different from the ‘critical wildlife habitat’. Since tigers are

13
Total Marks : 200
Online Prelims TEST - 20 (SUBJECT WISE)
( InsightsIAS Mock Test Series for UPSC Preliminary Exam 2020 ) Mark Scored : 0

territorial big cats, hence considering their social land tenure dynamics, the ‘core / critical tiger
habitat’ has been viewed separately from the ‘critical wildlife habitat’, which is applicable to other
wild animal species.

https://pib.gov.in/newsite/PrintRelease.aspx?relid=69806

22 Consider the following statements regarding Nonylphenol


1. It is a persistent, toxic, bio-accumulative chemical.
2. It is known to be an endocrine disrupting chemical which is hazardous to the environment and
human health
3. It is substantially used as a surfactant in the textile industry.

Which of the statements given above is/are correct?


A. 1 and 2 only

m
B. 2 and 3 only

o
C. 3 only

c
D. 1, 2 and 3

b .
u
Correct Answer : D

Answer Justification :

e h
te
All the statements given above are correct.

tu
Nonylphenol is a persistent, toxic, bio-accumulative chemical. Nonylphenol (NP) is known to be an

.
endocrine disrupting chemical which is hazardous to the environment and human health, besides

w
posing serious threat to aquatic life and other fauna.

w
The chemical has also been found to have a number of adverse reproductive and hormonal effects in
human beings and can cause carcinogenic effects on human body.

w
Nonylphenol has amphiphilic properties and is substantially used as a surfactant in the textile
industry.

23 Consider the following statements regarding Coastal Regulation Zones (CRZ)


1. CRZ norms have been issued under Water (Prevention & Control of Pollution) Act, 1974
2. CRZ IV includes ecologically sensitive areas, where no construction is allowed except activities for
atomic power plants, defence.

Which of the statements given above is/are correct?


A. 1 only
B. 2 only
C. Both 1 and 2
D. Neither 1 nor 2

14
Total Marks : 200
Online Prelims TEST - 20 (SUBJECT WISE)
( InsightsIAS Mock Test Series for UPSC Preliminary Exam 2020 ) Mark Scored : 0

Correct Answer : D

Answer Justification :

Both the statements given above are incorrect.

What is Coastal Regulation Zone?

The coastal zone is a transition area between marine and territorial zones. It includes shore
ecosystems, wetland ecosystems, mangrove ecosystems, mudflat ecosystems, sea grass ecosystems,
salt marsh ecosystems and seaweed ecosystems.

CRZ norms have been issued under Section 3 of the Environment Protection Act, 1986. Hence
Statement 1 is incorrect.

m
CRZ classified into four categories:

. co
CRZ-I: includes ecologically sensitive areas, where no construction is allowed except activities

b
for atomic power plants, defence. Hence Statement 2 is incorrect.

h u
CRZ-II: includes designated urban areas that are substantially built up. Construction activities

e
are allowed on the landward side only.

tu te
24 Abhor Wildlife Sanctuary, sometime seen in the news, is largely known for the conservation of

.
A. Indian Bison

w
B. Olive Ridley Turtle
C. Black Buck

w
D. Sangai Deer

w
Correct Answer : C

Answer Justification :

Abohar Wildlife Sanctuary (AWS)

1. The sanctuary is spread over thirteen villages, in Fazilka District of Punjab, home to
the Bishnoi community.

2. The sanctuary is also home to several other animals apart from the black buck and blue bull
such as Porcupines, wild boars and black ducks.

3. The Abohar Wildlife Sanctuary spreads over private land unlike others.

15
Total Marks : 200
Online Prelims TEST - 20 (SUBJECT WISE)
( InsightsIAS Mock Test Series for UPSC Preliminary Exam 2020 ) Mark Scored : 0

4. A unique initiative has been started by the Bishnoi community to protect the black buck
considered to be sacred by them.

25 Which of the followings are suitable ways to clean an oil spill?


1. Burning
2. Skimming
3. Through Pyrolysis
4. Usage of Chemicals

Select the correct answer using the code given below


A. 1, 2, 3 and 4
B. 1 and 4 only
C. 1 and 3 only
D. 2 and 3 only

o m
. c
Correct Answer : A

b
Answer Justification :

h u
An oil spill is the release of a liquid petroleum hydrocarbon into the environment, especially the

e
marine ecosystem, due to human activity, and is a form of pollution. Often oil spills are generally

te
caused by accidents involving tankers, pipelines, refineries, drilling rigs, barges etc. They can be
caused due to natural disasters like hurricanes. They can happen due to deliberate actions of

u
terrorists, anti-social elements and countries at war. Discharge of ballast, sludge and water used for

.t
cleaning of tanks. They can be caused by equipment breakdown or due to carelessness of people.

w w
Mainly there are three ways to clean an oil spill:

wThrough burning. This can lead to enormous atmospheric pollution. Hence, statement 1 is
correct.

Through Skimming. These cannot be effective if a huge area is affected. Hence, statement
2 is incorrect

By using chemicals to break the oil. But in this case the dispersants themselves can be
toxic. Hence, statement 3 is correct.

26 Consider the following statements regarding Micro Plastics


1. It is plastic material of size less than 1 mm
2. Micro plastic is emerging as a major threat to the entire ecosystem in general and marine
ecosystem in particular.

16
Total Marks : 200
Online Prelims TEST - 20 (SUBJECT WISE)
( InsightsIAS Mock Test Series for UPSC Preliminary Exam 2020 ) Mark Scored : 0

Which of the statements given above is/are correct?


A. 1 only
B. 2 only
C. Both 1 and 2
D. Neither 1 nor 2

Correct Answer : B

Answer Justification :

Micro plastic is plastic material of size less than 5 mm, originally generated by degeneration of used
plastic, cosmetic and personal industry etc. Hence, statement 1 is incorrect.

This is emerging as a major threat to the entire ecosystem in general and marine ecosystem in

m
particular. Micro plastic is often consumed by mammal, reducing actual food intake due to false

o
satiation, leading to starvation. Inability to digest this might lead to death. Respirational intake is

. c
fatal to both animals and humans causing suffocation. Its impact on marine life is really adverse. It
transfers toxin into the food chain, often reaching human bodies if they are at the top of it. Hence,

b
statement 2 is correct.

h u
27 With reference to Noise Pollution, consider the following statements

te e
1. A person's hearing can be damaged if exposed to noise levels over 75 dB over a prolonged period of
time.

u
2. The World Health Organization recommends that the sound level indoors should be less than 30 db.

t
3. Excessive noise has been registered as a crime under the Section 268 of IPC.

w .
Which of the statements given above is/are correct?
A. 2 only

w
B. 1 and 2 only

w
C. 3 only
D. 1, 2 and 3

Correct Answer : D

Answer Justification :

Noise pollution is the disturbing noise with harmful impact on the activity of human or animal life.
The source of outdoor noise worldwide is mainly caused by machines, transportation systems, motor
vehicles engines and trains. Outdoor noise is summarized by the word environmental noise. Sound
is measured in decibels (dB).

An increase of about 10 dB is approximately double the increase in loudness. A person's hearing can
be damaged if exposed to noise levels over 75 dB over a prolonged period of time. Hence,
statement 1 is correct.

The World Health Organization recommends that the sound level indoors should be less than 30 db.

17
Total Marks : 200
Online Prelims TEST - 20 (SUBJECT WISE)
( InsightsIAS Mock Test Series for UPSC Preliminary Exam 2020 ) Mark Scored : 0

Hence, statement 2 is correct.

Excessive noise has been registered as a crime under the Section 268 of IPC. Section 268 talks
about public nuisance and any person who is guilty of any public nuisance if that person commits an
act which can cause injury to any person, which may irritate the general public or which can cause
any general obstruction. Hence, statement 3 is correct.

28 Consider the following statements


1. Phytoremediation is use of plants to remove contaminants from soil and water.
2. All contaminants are easily treated by bioremediation.

Which of the statements given above is/are correct?


A. 1 only
B. 2 only

m
C. Both 1 and 2

o
D. Neither 1 nor 2

Correct Answer : A

b . c
Answer Justification :

h u
te e
Bioremediation is a treatment that uses naturally occurring organisms to break down
hazardous substances into less toxic or non-toxic substances. It uses microorganisms to

u
degrade organic contaminants in soil, groundwater, sludge, and solids. Bioremediation may be

t
conducted in situ or ex situ. It has been relied up on to clean oil spills in the recent past using the

.
bacteria of family Pseudomonas and other bacteria like Alcanivorax or Methylocella Silvestris.

w
Phytoremediation is use of plants to remove contaminants from soil and water. Hence, statement
1 is correct.

w w
Not all contaminants are easily treated by bioremediation using microorganisms. For example,
heavy metals such as cadmium and lead are not readily absorbed or captured by microorganisms.
Hence, statement 2 is incorrect.

29 With reference to the Air (prevention and control of pollution) act, 1981, consider the following
statements
1. The Act makes provisions for the establishing of Central Pollution Control Board (CPCB) at the apex
level and State Pollution Control Boards at the state level
2. According to this act the air pollutant means any solid, liquid or gaseous substance including noise.

Which of the statements given above is/are correct?


A. 1 only
B. 2 only
C. Both 1 and 2
D. Neither 1 nor 2

18
Total Marks : 200
Online Prelims TEST - 20 (SUBJECT WISE)
( InsightsIAS Mock Test Series for UPSC Preliminary Exam 2020 ) Mark Scored : 0

Correct Answer : C

Answer Justification :

Both the statements are correct.

The Air (Prevention and Control of Pollution) Act, 1981 aims to provide for the prevention, control
and abatement of air pollution, for the establishment, with a view to carrying out the aforesaid
purposes, of Boards, for conferring on and assigning to such Boards powers and functions relating
thereto and for matters connected therewith. The Act makes provisions for the establishing of
Central Pollution Control Board (CPCB) at the apex level and State Pollution Control
Boards at the state level. Hence, statement 1 is correct.

according to this act, the “air pollutant” means any solid, liquid or gaseous substance (including
noise) present in the atmosphere in such concentration as may be or tend to be injurious to human

m
beings or other living creatures or plants or property or environment. Hence, statement 2 is

o
correct.

30 Which of the following bio plastic used for making bank notes?

b . c
A.
B.
Polyhydroxyalkanoates (PHA)
Polyhydroxybutyrate (PHB)

h u
e
C. Polyamide 11 (PA 11)

te
D. Polylactic Acid (PLA)

Correct Answer : B

.tu
w w
Answer Justification :

Bio plastics are developed as an alternative to the petroleum-based plastics. Increased use of

w
petroleum-based plastics is associated with producing more greenhouse gases. Polyhydroxybutyrate
(PHB) is used for making bank notes and car parts etc. Polyamide 11 (PA 11) prepared from
vegetable oils is used for making oil and gas flexible pipes, and electrical anti-termite cable
sheathing etc. Hence, option B is correct.

31 Which of the following is/are In-situ bioremediation techniques?


1. Bioventing
2. Biosparging
3. Bioreactors
4. Bioagumentation

Which of the statements given above is/are correct?


A. 1 and 2 only
B. 2 and 3 only
C. 1, 2 and 4 only
D. 1, 2, 3 and 4 only

19
Total Marks : 200
Online Prelims TEST - 20 (SUBJECT WISE)
( InsightsIAS Mock Test Series for UPSC Preliminary Exam 2020 ) Mark Scored : 0

Correct Answer : C

Answer Justification :

Bioremediation is a process used to treat contaminated media, including water, soil and subsurface
material, by altering environmental conditions to stimulate growth of microorganisms and degrade
the target pollutants. In many cases, bioremediation is less expensive and more sustainable than
other remediation alternatives. Biological treatment is a similar approach used to treat wastes
including wastewater, industrial waste and solid waste.

In situ bioremediation

It involves treatment of the contaminated material at the site.

o m
Bioventing: supply of air and nutrients through wells to contaminated soil to stimulate the

c
growth of indigenous bacteria. It is used for simple hydrocarbons and can be used where the

.
contamination is deep under the surface. Hence, statement 1 is correct.

u b
Biosparging: Injection of air under pressure below the water table to increase groundwater

h
oxygen concentrations and enhance the rate of biological degradation of contaminants by

e
naturally occurring bacteria. Hence, statement 2 is correct.

u te
Bioaugmentation: Microorganisms are imported to a contaminated site to enhance

t
.
degradation process. Hence, statement 4 is correct.

w
32 Consider the following statements regarding Eutrophication

w
1. It leads to increase in the primary productivity of the water body.

w
2. It can occur outside water bodies also.

Which of the statements given above is/are correct?


A. 1 only
B. 2 only
C. Both 1 and 2
D. Neither 1 nor 2

Correct Answer : C

Answer Justification :

Eutrophication, or hypertrophication, is when a body of water becomes overly enriched with


minerals and nutrients which induce excessive growth of algae. This process may result in oxygen
depletion of the water body. It leads to increase in the primary productivity of the water body or
'bloom' of phytoplankton. Hence, statement 1 is correct.

20
Total Marks : 200
Online Prelims TEST - 20 (SUBJECT WISE)
( InsightsIAS Mock Test Series for UPSC Preliminary Exam 2020 ) Mark Scored : 0

Eutrophication can also occur outside water bodies. For example, soils can be eutrophic when they
have high levels of nitrogen, phosphorous or other nutrients. Hence, statement 2 is correct.

33 Consider the following statements


1. Ocean acidification has fastened after the Industrialization.
2. Ocean acidification increases the concentration of carbonate.

Which of the statements given above is/are correct?


A. 1 only
B. 2 only
C. Both 1 and 2
D. Neither 1 nor 2

m
Answer Justification :

Both the statements given above are correct.

. co
b
Ocean acidification is the ongoing decrease in the pH of the Earth's oceans, caused by the uptake of

u
carbon dioxide from the atmosphere. Seawater is slightly basic, and ocean acidification involves a

h
shift towards pH-neutral conditions rather than a transition to acidic conditions.

e
The Ocean acidification has increased only after the industrialization. Pre-Industrialization pH value

te
of the Ocean water was 8.179 which were coming down to 8.1074 which constituted 19% increase
of H+ ions. Hence, statement 1 is correct.

.tu
It reduces the concentration of carbonate, which is important for building block in seawater.

w
Hence, statement 2 is incorrect.

w w
34 Consider the following statements regarding Montreal Protocol
1. Under this treaty many countries of the world have agreed to phase out of ozone-depleting
substances.
2. It is the first international treaty to have achieved universal ratification in the history of United
Nations.

Which of the statements given above is/are correct?


A. 1 only
B. 2 only
C. Both 1 and 2
D. Neither 1 nor 2

Correct Answer : C

Answer Justification :

21
Total Marks : 200
Online Prelims TEST - 20 (SUBJECT WISE)
( InsightsIAS Mock Test Series for UPSC Preliminary Exam 2020 ) Mark Scored : 0

Montreal Protocol is an international treaty signed in 1987 after the Vienna Convention and came
into force in 1989. According to this treaty, many countries of the world have agreed to phase out of
ODSs. Hence, statement 1 is correct.

Montreal Protocol is the first international treaty to have achieved universal ratification in the
history of United Nations. It is also highly successful international arrangement, as it has phased-
out more than 95% of the ODS so far as per its main mandate (CFCs) in less than 30 years of its
existence. Hence, statement 2 is correct.

35 Consider the following statements


1. Plankton are the producers present in the aquatic ecosystem.
2. They are active swimmers and typically flow with the water current.

Which of the statements given above is/are correct?

m
A. 1 only

o
B. 2 only

c
C. Both 1 and 2

.
D. Neither 1 nor 2

u b
h
Correct Answer : A

Answer Justification :

te e
u
Plankton are the diverse collection of organisms that live in large bodies of water. They are the

t
producers present in the aquatic ecosystem. Hence, statement 1 is correct.

w .
They are passive swimmers due to their limited locomotory power. They typically flow with the
water current. For example, phytoplankton, zooplankton (protozoa), mycoplankton (fungi) and some

w
large animals like Jellyfish. Some organisms like sea urchins and starfish, begin their life as

w
plankton and later on become benthic or nektons, they are termed as Meroplankton. Hence,
statement 2 is incorrect.

36 Which of the following factors limiting the productivity of aquatic habitats


1. Sunlight
2. Nutrient levels
3. Dissolved Oxygen
4. Temperature
5. Salinity

Which of the statements given above is/are correct?


A. 1, 2, 4 and 5 only
B. 1, 2, 3 and 4 only
C. 1, 3, 4 and 5 only
D. 1, 2, 3, 4 and 5

22
Total Marks : 200
Online Prelims TEST - 20 (SUBJECT WISE)
( InsightsIAS Mock Test Series for UPSC Preliminary Exam 2020 ) Mark Scored : 0

Correct Answer : D

Answer Justification :

Natural aquatic habitats include ponds, lakes, rivers, streams, springs, estuaries, bays, and various
types of wetlands. Some of these habitats are shallow and others deep, some are cold- water and
others warm-water, some are freshwater and others saltwater, and some have high oxygen levels
and others little oxygen.

Factors Limiting the Productivity of Aquatic Habitats

Limiting factors can suppress the productivity of an ecosystem because there is too much or too
little of it. Major limiting factors in an aquatic ecosystem are as follows: Hence, all the above
statements are correct.

Sunlight

o m
Nutrient levels

b . c
Dissolved Oxygen

h u
Temperature

te e
Salinity

.tu
w w
37 Consider the following statement regarding National Mission on Sustainable Habitat
1. It is governed by the Ministry of New and Renewable Energy.

w
2. It aims to make urban areas more climate friendly and less susceptible to climate change

Which of the statements given above is/are correct?


A. 1 only
B. 2 only
C. Both 1 and 2
D. Neither 1 nor 2

Correct Answer : B

Answer Justification :

The National Mission on Sustainable Habitat is one of the eight climate mission of the Government
of India under the NAPCC mitigation strategy.

It plans to make urban areas more climate friendly and less susceptible to climate change by a

23
Total Marks : 200
Online Prelims TEST - 20 (SUBJECT WISE)
( InsightsIAS Mock Test Series for UPSC Preliminary Exam 2020 ) Mark Scored : 0

multi-pronged approach to mitigate and adapt to it. Hence, statement 2 is correct.

It is governed by the Ministry of Urban Development. Hence, statement 1 is incorrect.

38 Which of the following is/are outcome of the Earth Summit, 1992?


1. Kyoto Protocol
2. Agenda 21
3. Forest Principles

Select the correct answer using the given below:


A. 1 only
B. 2 only
C. 2 and 3 only
D. 1 and 3 only

o m
c
Correct Answer : C

Answer Justification :

b .
h u
United Nations Conference On Environment And Development (UNCED), also known as the Rio

e
Summit, Rio Conference, Earth Summit held in Rio de Janeiro in June 1992.

te
The Earth Summit resulted in the following documents:

u
• Rio Declaration on Environment and Development

t
• Agenda 21

.
• Forest Principles

w
Hence, option (c) is correct.

w w
39 With reference to the Cartagena Protocol on Biosafety, Consider the following statements:
1. It is an additional agreement to the Convention on Biological Diversity (CBD).
2. The Protocol sets the deadline to prohibit the import and export of Living Modified Organisms
(LMO) from one country to another.

Which of the statements given above is/are correct?


A. 1 only
B. 2 only
C. Both 1 and 2
D. Neither 1 nor 2

Correct Answer : A

Answer Justification :

Biosafety refers to the need to protect human health and the environment from the possible adverse

24
Total Marks : 200
Online Prelims TEST - 20 (SUBJECT WISE)
( InsightsIAS Mock Test Series for UPSC Preliminary Exam 2020 ) Mark Scored : 0

effects of the products of modern biotechnology.

The Cartagena Protocol on Biosafety is an additional agreement to the Convention on Biological


Diversity. Hence, statement 1 is correct.
The Protocol establishes procedures for regulating the import and export of LMOs from one
country to another.

The Protocol also requires Parties to ensure that LMOs being shipped from one country to
another are handled, packaged and transported in a safe manner.

The shipments must be accompanied by documentation that clearly identifies the LMOs,
specifies any requirements for the safe handling, storage, transport and use and provides
contact details for further information. . Hence, statement 2 is incorrect.

m
40 Which of the following sites of India is/are present in Montreux Record?

o
1. Loktak Lake

c
2. Keoladeo National Park

.
3. Kolleru lake

Select the correct answer using the code given below:


A. 1 only

u b
e h
B. 1 and 2 only
C. 3 only

te
D. 1, 2 and 3

Correct Answer : B

.tu
w w
Answer Justification :

The Montreux Record is a register of wetland sites on the List of Wetlands of International

w
Importance where changes in ecological character have occurred, are occurring, or are likely to
occur as a result of technological developments, pollution or other human interference.

Keoladeo National Park, Rajasthan and Loktak Lake, Manipur have been included in
Montreux Record in 1990 and in 1993 respectively.

Chilika Lake, Orissa included in Montreux Record in 1993 have been removed in
November 2002.

41 The Changwon Declaration was adopted by:

A. Ramsar Convention on Wetlands


B. Convention on International Trade in Endangered Species of Fauna and Flora (CITES)
C. The Basel Convention on the Control of Trans boundary Movements of Hazardous Wastes and
Their Disposal
D. Stockholm Convention on Persistent Organic Pollutants

25
Total Marks : 200
Online Prelims TEST - 20 (SUBJECT WISE)
( InsightsIAS Mock Test Series for UPSC Preliminary Exam 2020 ) Mark Scored : 0

Correct Answer : A

Answer Justification :

The Changwon Declaration on human well-being and wetlands ighlights positive action for ensuring
human well-being and security in the future under the themes - water, climate change, people’s
livelihood and health, land use change, and biodiversity.

It was adopted by Ramsar convention in 2008.

Hence, option (a) is correct.

42 With reference to the Convention on International Trade in Endangered Species of Wild Fauna and
Flora (CITES), consider the following statements:

m
1. It is administered through the United Nations Environment Programme (UNEP).

o
2. Appendix I of the convention includes species threatened with extinction and provides the greatest

. c
level of protection, including restrictions on commercial trade.

b
3. Appendix III of the convention includes species for which a range country has asked other Parties to

u
help in controlling international trade.

h
Which of the statements given above is/are correct?

e
A. 1 and 2 only

te
B. 2 and 3 only
C. 1 and 3 only

tu
D. 1, 2 and 3

Correct Answer : D

w .
w w
Answer Justification :

All the statements given above are correct.

The Convention on International Trade in Endangered Species of Wild Fauna and Flora
(CITES) is an international agreement between governments entered into force in 1975,
and became the only treaty to ensure that international trade in plants and animals does not
threaten their survival in the wild.

CITES is administered through the United Nations Environment Programme (UNEP). A


Secretariat, located in Geneva, Switzerland, oversees the implementation of the treaty and assists
with communications between
countries

Appendix I of the convention includes species threatened with extinction and provides the
greatest level of protection, including restrictions on commercial trade. Examples include
gorillas, sea turtles, most lady slipper orchids, and giant pandas.

26
Total Marks : 200
Online Prelims TEST - 20 (SUBJECT WISE)
( InsightsIAS Mock Test Series for UPSC Preliminary Exam 2020 ) Mark Scored : 0

Appendix III of the convention includes species for which a range country has asked other
Parties to help in controlling international trade. Examples include map turtles, walruses and
Cape stag beetles.

43 Consider the following statements regarding Animal Welfare Board of India


1. It is a statutory body established in 1962, under the Prevention of cruelty to animals act, 1960
2. The board has 28 members, each with a tenure of three years.
3. It ensures that the animal welfare laws are strictly adhered to and provides grants as well
as recognition to Animal Welfare Organisations

Which of the statements given above is/are correct?


A. 1 and 2 only
B. 2 only
C. 2 and 3 only

m
D. 1, 2 and 3

Correct Answer : D

. co
Answer Justification :

u b
All the statements given above are correct.

e h
te
About Animal Welfare Board of India

.tu
w w
It is a statutory body established in 1962, under the Prevention of cruelty to animals act, 1960

wIt is an advisory body on animal welfare laws and promotes animal welfare in the country.

It ensures that the animal welfare laws are strictly adhered to and provides grants as well as
recognition to Animal Welfare Organisations

It also advises the government of India on animal welfare issues.

The board has 28 members, each with a tenure of three years. Several government
organisations, along with animal rights activists and parliamentarians, are represented on the
Board.

44 Consider the following statements regarding Houbara Bustards


1. Houbara Bustards are terrestrial birds that belong to several species, including some of the largest

27
Total Marks : 200
Online Prelims TEST - 20 (SUBJECT WISE)
( InsightsIAS Mock Test Series for UPSC Preliminary Exam 2020 ) Mark Scored : 0

flying birds.
2. The population of the Asian houbara bustards extends from northeast Asia, across central Asia, the
Middle East, and the Arabian Peninsula to reach the Sinai desert.

Which of the statements given above is/are correct?


A. 1 only
B. 2 only
C. Both 1 and 2
D. Neither 1 nor 2

Correct Answer : C

Answer Justification :

m
Both the statements are correct.

Houbara bustards

. co
b
Bustards are large, terrestrial birds that belong to several species, including some of the largest

u
flying birds.

e h
The houbara bustard, which lives in arid climates, comes in two distinct species as recognized by

te
the International Union for Conservation of Nature,residing in:

tu
North Africa (Chlamydotis undulate)

w .
Asia (Chlamydotis macqueenii).

w w
The population of the Asian houbara bustards extends from northeast Asia, across central Asia, the
Middle East, and the Arabian Peninsula to reach the Sinai desert.

45 India's first garbage festival was organized in which state?

A. Madhya Pradesh
B. Mizoram
C. Karnataka
D. Chhattisgarh

Correct Answer : D

Answer Justification :

India's 1st Garbage Festival Organized in Chhattisgarh. The festival aimed to increase awareness,
use the waste generated in the city creatively, and to showcase multiple techniques for reusing

28
Total Marks : 200
Online Prelims TEST - 20 (SUBJECT WISE)
( InsightsIAS Mock Test Series for UPSC Preliminary Exam 2020 ) Mark Scored : 0

things that are labeled as garbage. Hence, option D is correct.

46 Consider the following statements regarding Temperate Deciduous Biome


1. Soils of temperate forests are podzolic and fairly deep
2. The trees shed their leaves in the summer season.

Which of the statements given above is/are correct?


A. 1 only
B. 2 only
C. Both 1 and 2
D. Neither 1 nor 2

Correct Answer : A

Answer Justification :

o m
. c
Temperate deciduous or temperate broad-leaf forests are a variety of temperate forest 'dominated'

b
by trees that lose their leaves each year. They are found in areas with warm moist summers and

u
cool winters. Soils of temperate forests are podozolic and fairly deep. Hence, statement 1 is

h
correct.

te e
The trees shed their leaves in the cold season. This is an adaptation for protecting themselves
against the winter snow and frost. Hence, statement 2 is incorrect.

.tu
47 The word Biodiversity was popularized by

A.
B.

w
Norman Myers
w
Edward Wilson

w
C. Ernst Haeckel
D. Arthur Tansley

Correct Answer : A

Answer Justification :

Edward Osborne Wilson (born June 10, 1929), usually cited as E. O. Wilson, is an American
biologist, naturalist, and writer. His biological specialty is myrmecology, the study of ants, on which
he has been called the world's leading expert.

Wilson has been called "the father of sociobiology" and "the father of biodiversity"[5] for his
environmental advocacy. Among his greatest contributions to ecological theory is the theory of
island biogeography and this theory served as the foundation of the field of conservation area
design, as well as the unified neutral theory of biodiversity of Stephen Hubbell.

29
Total Marks : 200
Online Prelims TEST - 20 (SUBJECT WISE)
( InsightsIAS Mock Test Series for UPSC Preliminary Exam 2020 ) Mark Scored : 0

48 Consider the following with reference to Specie richness


1. Alpha Diversity refers to the diversity within a particular area or ecosystem and is usually expressed
by the number of species (i.e., species richness) in that ecosystem.
2. Beta Diversity is a measure of the overall diversity for the different ecosystems within a region.
3. Gamma Diversity is a comparison of diversity between ecosystems, usually measured as the change
in the amount of species between the ecosystems.

Which of the statements given above is/are correct?


A. 1 only
B. 2 and 3 only
C. 1 and 3 only
D. 1, 2 and 3

m
Correct Answer : A

Answer Justification :

. co
b
Species richness is the measure of number of species found in a community.

u
a) Alpha diversity: It refers to the diversity within a particular area or ecosystem, and is usually

h
expressed by the number of species (i.e., species richness) in that ecosystem.

te e
b) Beta diversity: It is a comparison of diversity between ecosystems, usually measured as the
change in amount of species between the ecosystems.

.tu
c) Gamma diversity: It is a measure of the overall diversity for the different ecosystems within a
region. Hence statement 2 and 3 are incorrect.

w w
49 Arrange the type of Plant species (including algae, fungi and lichens) in decreasing order of their
occurrence in India with respect to the world
1. Fungi

w
2. Pteridophytes
3. Gymnosperms
4. Algae

Which of the following order is correct?


A. 1,4,2,3
B. 1,4,3,2
C. 4,1,2,3
D. 4,1,3,2

Correct Answer : C

Answer Justification :

Total number of plant species (including virus, bacteria, algae, fungi and lichens) and their status in

30
Total Marks : 200
Online Prelims TEST - 20 (SUBJECT WISE)
( InsightsIAS Mock Test Series for UPSC Preliminary Exam 2020 ) Mark Scored : 0

World and India

Number of known Species Percentage of


SI. No. Type Occurrence in
World India India

I Flowering Plants
1. Gymnosperms 1021 74 7.35%
2. Angiosperms 268600 18043 6.72%

III Non-flowering Plants

16236 2523 15.54%


1. Bryophytes
12000 1267 10.57%
2. Pteridophytes

m
III Others
1. Virus and Bacteria 11813 986 8.77%

o
2. Algae 40000 7284 18.21%

. c
3. Fungi 98998 14883 15.09%
4. Lichens 17000 2401 14.12%

u b
h
Total 465668 47513 –

te e
50 Consider the following statements regarding Leatherback Sea Turtle
1. It is the largest of all living turtles.

u
2. The geographical distribution is limited to warm tropical waters.

.t
Which of the statements given above is/are correct?

w
A. 1 only
B. 2 only

w
C. Both 1 and 2

w
D. Neither 1 nor 2

Correct Answer : A

Answer Justification :

31
Total Marks : 200
Online Prelims TEST - 20 (SUBJECT WISE)
( InsightsIAS Mock Test Series for UPSC Preliminary Exam 2020 ) Mark Scored : 0

o m
b . c
u
The Leatherback Sea Turtle, sometimes called the lute turtle, is the largest of all living turtles.

h
The leatherback turtle is a species with a cosmopolitan global range, reaching as far north as

e
Alaska and Norway and as far south as Cape Agulhas in Africa and the southernmost tip of

te
New Zealand. The leatherback is found in all tropical and subtropical oceans, and its range
extends well into the Arctic Circle. Leatherback turtles can be found primarily in the open ocean.

tu
Leatherbacks feed on jellyfish prey throughout the day, preferring deeper water in the daytime and

.
shallower water at night.

w w
51 Consider the following statements regarding Invasive species
1. All alien species introduced to a region are invasive in nature.

w
2. According to the Convention for Biological Diversity, invasive alien species are the second largest
cause of biodiversity loss in the world.
3. Invasive Species Specialist Group is organized by IUCN’s Species Survival Commission.

Which of the statements given above is/are correct?


A. 1 and 2 only
B. 2 and 3 only
C. 1 and 3 only
D. 1, 2 and 3

Correct Answer : B

Answer Justification :

An alien plant that has escaped from its original ecosystem and is reproducing on its own in the
regional flora is considered a naturalized species. Those naturalized aliens that become so

32
Total Marks : 200
Online Prelims TEST - 20 (SUBJECT WISE)
( InsightsIAS Mock Test Series for UPSC Preliminary Exam 2020 ) Mark Scored : 0

successful as to spread in the flora and displace native biota or threatens valued
environmental, agricultural or personal resources by the damage it causes are considered
invasive.

According to the Convention for Biological Diversity, invasive alien species are the second largest
cause of biodiversity loss in the world and impose high costs to agriculture, forestry, and aquatic
ecosystems.

The IUCN’s Species Survival Commission organizes the Invasive Species Specialist Group,
which is a global network of science and policy experts. The Group aims to reduce threats to natural
systems and native species by increasing the awareness of IAS and examining ways to prevent,
control or eradicate them.

52 Consider the following statements regarding Conservation reserves

m
1. They act as buffer zones to or connectors and migration corridors between established national

o
parks, wildlife sanctuaries etc.

c
2. These protected area categories were first introduced Environment Protection Act, 1986.

.
3. Tiruppadaimarathur conservation reserve in Tamil Nadu is the first Conservation Reserve in the

b
country.

Which of the statements given above is/are correct?

h u
e
A. 1 and 2 only

te
B. 2 and 3 only
C. 1 and 3 only

u
D. 1, 2 and 3

.t
w
Correct Answer : C

w
Answer Justification :

w
Conservation reserves and community reserves in India are terms denoting protected areas of
India which typically act as buffer zones to or connectors and migration corridors between
established national parks, wildlife sanctuaries and reserved and protected forests of India.

Such areas are designated as conservation areas if they are uninhabited and completely owned by
the Government of India but used for subsistence by communities and community areas if part of
the lands is privately owned.

These protected area categories were first introduced in the Wildlife (Protection) Amendment
Act of 2002 − the amendment to the Wildlife Protection Act of 1972.

Tiruppadaimarathur conservation reserve near Thirunelveli District of Tamil Nadu, declared in


2005, is the first Conservation Reserve in the country.

33
Total Marks : 200
Online Prelims TEST - 20 (SUBJECT WISE)
( InsightsIAS Mock Test Series for UPSC Preliminary Exam 2020 ) Mark Scored : 0

53 Consider the following pairs


National Parks State
1. Dudhwa National Park Uttar Pradesh
2. Indira Gandhi National Park Maharashtra
3. Desert National Park Gujarat
4. Phawngpui Blue Mountain National Park Mizoram

Which of the pairs given above is/are correct?


A. 1 and 2 only
B. 2 and 3 only
C. 3 and 4 only
D. 1 and 4 only

Correct Answer : D

o m
Answer Justification :

b . c
u
Indira Gandhi National Park is located in Tamil Nadu.

Desert National Park is located in Rajasthan.

e h
te
http://www.wiienvis.nic.in/Database/npa_8231.aspx

.tu
54 The criteria to qualify as a biodiversity hotspot is/are

w
1. It must have at least 1,500 vascular plants as endemics
2. It must have lost at least 70 percent of their original habitat.

w w
Which of the statements given above is/are correct?
A. 1 only
B. 2 only
C. Both 1 and 2
D. Neither 1 nor 2

Correct Answer : C

Answer Justification :

The British biologist Norman Myers coined the term "biodiversity hotspot" in 1988 as a
biogeographic region characterized both by exceptional levels of plant endemism and by serious
levels of habitat loss. In 1990 Myers added a further eight hotspots, including four Mediterranean-
type ecosystems. Conservation International (CI) adopted Myers’ hotspots as its institutional
blueprint in 1989, and in 1996, the organization made the decision to undertake a reassessment of
the hotspots concept. Three years later an extensive global review was undertaken, which
introduced quantitative thresholds for the designation of biodiversity hotspots.

34
Total Marks : 200
Online Prelims TEST - 20 (SUBJECT WISE)
( InsightsIAS Mock Test Series for UPSC Preliminary Exam 2020 ) Mark Scored : 0

According to Conservation International, to qualify as a biodiversity hotspot, a region must meet


two strict criteria:

It must have at least 1,500 vascular plants as endemics — which is to say, it must have a
high percentage of plant life found nowhere else on the planet. A hotspot, in other words, is
irreplaceable.

It must have 30% or less of its original natural vegetation. In other words, it must be
threatened.

55 Consider the following statements regarding Wildlife Sanctuaries


1. The Wild Life (Protection) Act, 1972 provides for the declaration of certain areas by the State

m
Government as Wildlife Sanctuary.
2. Wildlife sanctuary can be created for a particular species.

o
3. Certain activities such as grazing of livestock, are prohibited in Wildlife Sanctuaries.

Which of the statements given above is/are correct?

b . c
u
A. 1 and 2 only
B. 2 and 3 only

h
C. 1 and 3 only

e
D. 1, 2 and 3

Correct Answer : A

tu te
w .
Answer Justification :

w
The Wild Life (Protection) Act of 1972 provided for the declaration of certain areas by the State
Government as wildlife sanctuaries if the area was thought to be of adequate ecological,

w
geomorphological and natural significance.

Certain activities such as grazing of livestock are regulated in sanctuaries and are prohibited in
National Parks.

Wildlife sanctuary can be created for a particular species whereas the national park is not primarily
focused on a particular species. Example Ranebennur Blackbuck Sanctuary, Karnataka.

56 Consider the following with respect to Biological Diversity Act, 2002


1. It was enacted by government to give effect the Convention on Biological Diversity (CBD)
2. It has provided for three tier mechanism at National, State and Local level to give effect this act.
3. The Chairperson shall be an eminent person having adequate knowledge and experience in the field
biological diversity.

Which of the statements given above is/are correct?


A. 1 only

35
Total Marks : 200
Online Prelims TEST - 20 (SUBJECT WISE)
( InsightsIAS Mock Test Series for UPSC Preliminary Exam 2020 ) Mark Scored : 0

B. 1 and 2 only
C. 2 and 3 only
D. 1, 2 and 3

Correct Answer : D

Answer Justification :

India enacted Biological Diversity Act in 2002 for giving effect to the provisions of the CBD.
Objective of this act is to regulate the access to genetic resources and protection of biodiversity.

It sets up National Biodiversity Authority (NBA), State Biodiversity Board (SBB) and
Biodiversity Management Committees at national, state and local level respectively.

m
The Chairperson shall be an eminent person having adequate knowledge and experience in

o
the conservation and sustainable use of biological diversity and in matters relating to

c
equitable sharing of benefits, to be appointed by the Central Government.

b .
u
57 Which of the following is/are methods of In situ conservation?

h
1. National parks and sanctuaries
2. Biosphere reserves

e
3. Preservation plots

te
4. Botanical garden

tu
Which of the options is/are correct?

.
A. 1 and 2 only

w
B. 1, 2 and 3 only
C. 3 and 4 only

w
D. 1, 2, 3 and 4

w
Correct Answer : B

Answer Justification :

In situ conservation

Conserving the animals and plants in their natural habitats is known as in situ conservation. This
includes the establishment of

National parks and sanctuaries

Biosphere reserves

Nature reserves

36
Total Marks : 200
Online Prelims TEST - 20 (SUBJECT WISE)
( InsightsIAS Mock Test Series for UPSC Preliminary Exam 2020 ) Mark Scored : 0

Reserved and protected forests

Preservation plots

Reserved forests

Botanical garden is method of Ex situ conservation.

58 This National park has an elevation of 1,829 metres and has been declared as Mixed UNESCO World
Heritage site. The vegetation of the park includes temperate broadleaf and mixed forests, Alpine
grasses and shrubs at higher altitudes. The park contains mammal species such as musk deer, snow
leopard, Himalayan tahr, wild dog, sloth bear, red panda, Tibetan wild ass, as well as reptiles

m
including rat snake and Russell's viper.

Which of the following options is the correct National park?


A. Great Himalayan National Park

. co
b
B. Nanda Devi and Valley of Flowers National Parks

u
C. Kaziranga National Park

h
D. Kanchenjunga National Park

Correct Answer : D

te e
Answer Justification :

.tu
w
The Kanchenjunga Park is situated in the North and West Sikkim districts in the Indian state of
Sikkim. It has an elevation of 1,829 metres (6,001 ft) to over 8,550 metres (28,050 ft) and has an

w
area of 849.50 km2 (327.99 sq mi). It is one of the few high-altitude National parks of India and was

w
recently included as a mixed-criteria UNESCO World Heritage site.

The vegetation of the park includes temperate broadleaf and mixed forests consisting of oaks, fir,
birch, maple, willow.[5] The vegetation of the park also includes Alpine grasses and shrubs at higher
altitudes along with many medicinal plants and herbs.

The park contains many mammal species including musk deer, snow leopard, Himalayan tahr, wild
dog, sloth bear, civet, Himalayan black bear, red panda, Tibetan wild ass, Himalayan blue sheep,
serow, goral and takin, as well as reptiles including rat snake and Russell's viper.

59 Consider the following pairs


Sacred Groves State
1. Kovil Kadu Karnataka
2. Ki Law Lyngdoh Meghalaya
3. Deorai Goa
4. Bugyal Himachal Pradesh

37
Total Marks : 200
Online Prelims TEST - 20 (SUBJECT WISE)
( InsightsIAS Mock Test Series for UPSC Preliminary Exam 2020 ) Mark Scored : 0

Which of the pairs given above is/are correct?


A. 1 and 2 only
B. 2 and 3 only
C. 3 and 4 only
D. 1 and 4 only

Correct Answer : B

Answer Justification :

List of Sacred Groves of India

State Local term for Sacred Groves


Andhra Pradesh Pavithravana

Arunachal Pradesh
Gumpa Forests (Attached to Buddhist
monasteries)

o m
. c
Goa Deorai, Pann

b
Jharkhand Sarana

u
Karnataka Devara Kadu

h
Kerala Kavu, Sara Kavu

e
Maharashtra Devrai, Devrahati, Devgudi

te
Manipur Gamkhap, Mauhak (Sacred bamboo Reserve)
Meghalaya Ki Law Lyngdoh, Ki Law Kyntang, Ki Law Niam

tu
Orissa Jahera, Thukuramma

.
Puducherry Kovil Kadu

w
Rajasthan Orans, Kenkris, Jogmaya

w
Tamil Nadu Swwami shoal, Koikadu
Uttarakhand Deo Bhumi, Bugyal (Sacred Alpine Meadows)

w
Garamthan, Harithan, Jahera, Sabitrithan,
West Bengal
Santalburithan

60 Consider the following statements:


1. Belmont Forum provides an opportunity to identify, study and deliver international environmental
research priorities.
2. The Ministry of Environment, Forest and Climate Change (MoEFCC) represents India in the
Belmont Forum

Which of the statements given above is/are correct?


A. 1 only
B. 2 only
C. Both 1 and 2
D. Neither 1 nor 2

Correct Answer : A

38
Total Marks : 200
Online Prelims TEST - 20 (SUBJECT WISE)
( InsightsIAS Mock Test Series for UPSC Preliminary Exam 2020 ) Mark Scored : 0

Answer Justification :

Belmont Forum is created in 2009. It is a high level group of the world's major funders of global
environmental change research and international
science councils. It provides an opportunity to identify, study and deliver
international environmental research priorities. Hence, statement 1 is correct.

The Ministry of Earth Science represents India in the Belmont Forum since 2012. Hence,
statement 2 is incorrect.

61 Which of the following gases is/are included in Montreal Protocol for phase out?
1. Hydrobromofluorocarbons (HBFCs)
2. Carbontetrachloride (CCl4)
3. Methylbromide (CH3Br)

Select the correct answer using the code given below:

o m
c
A. 1 and 2 only

.
B. 2 and 3 only

b
C. 1 and 3 only

u
D. 1, 2 and 3

e h
te
Correct Answer : D

u
Answer Justification :

.t
All the above given gases are under Montreal protocol for gradual phase out.

w
The Montreal Protocol on Substances that Deplete the Ozone Layer is the landmark multilateral

w
environmental agreement that regulates the production and consumption of nearly 100 man-made
chemicals referred to as ozone depleting substances (ODS). When released to the atmosphere,

w
those chemicals damage the stratospheric ozone layer, Earth’s protective shield that protects
humans and the environment from harmful levels of ultraviolet radiation from the sun. Adopted on
15 September 1987, the Protocol is to date the only UN treaty ever that has been ratified every
country on Earth - all 197 UN Member States.

62 With reference to the International Tropical Timber Organization (ITTO), consider the following
statements:
1. It is an intergovernmental organization.
2. ITTO’s membership represents about 50% of the global tropical timber trade.

Which of the statements given above is/are correct?


A. 1 only
B. 2 only
C. Both 1 and 2
D. Neither 1 nor 2

39
Total Marks : 200
Online Prelims TEST - 20 (SUBJECT WISE)
( InsightsIAS Mock Test Series for UPSC Preliminary Exam 2020 ) Mark Scored : 0

Correct Answer : A

Answer Justification :

The International Tropical Timber Organization (ITTO) is an intergovernmental


organization promoting the sustainable management and conservation of tropical forests and the
expansion and diversification of international trade in tropical timber from sustainably managed
and legally harvested forests. Hence, statement 1 is correct.

ITTO’s membership represents about 90% of the global tropical timber trade and more than
80% of the world’s tropical forests. Hence, statement 2 is incorrect.

https://www.itto.int/about_itto/

m
63 Which of the following is/are covered as Short Lived Climate Pollutants (SLCP) under Climate and

o
Clean Air Coalition (CCAC)?

. c
1. Brown Carbon

b
2. Methane (CH4)

u
3. Hydroflurocarbons (HFCs)
4. Tropospheric Ozone

e
Select the correct answer using the code given below:
h
te
A. 1, 2 and 3 only
B. 2, 3 and 4 only

tu
C. 1 and 4 only

.
D. 1, 2, 3 and 4

w
Correct Answer : B
w
w
Answer Justification :

The Climate and Clean Air Coalition is a voluntary partnership of governments, intergovernmental
organizations, businesses, scientific institutions and civil society organizations committed to
improving air quality and protecting the climate through actions to reduce short-lived climate
pollutants.

Our global network currently includes over 120 state and non-state partners, and hundreds of local
actors carrying out activities across economic sectors.

Short Lived Climate Pollutants (SLCP) under Climate and Clean Air Coalition (CCAC) are:

1. Black Carbon

2. Methane (CH4)

3. Hydroflurocarbons (HFCs)

40
Total Marks : 200
Online Prelims TEST - 20 (SUBJECT WISE)
( InsightsIAS Mock Test Series for UPSC Preliminary Exam 2020 ) Mark Scored : 0

4. Tropospheric Ozone

Hence, option (b) is correct.

64 Consider the following with reference to Traditional Knowledge Digital Library


1. It is a collaboration between Council of Scientific and Industrial Research (CSIR) and Ministry of
AYUSH.
2. Access of TKDL has been provided to International Patent offices (IPOs) to prevent wrong patents.

Which of the statements given above is/are correct?


A. 1 only
B. 2 only
C. Both 1 and 2
D. Neither 1 nor 2

o m
c
Correct Answer : C

Answer Justification :

b .
h u
The Ministry of AYUSH had established Traditional Knowledge Digital Library (TKDL) in

e
collaboration with Council for Scientific & Industrial Research (CSIR). As per the

te
information provided by the CSIR, TKDL consisting of more than 2.90 lakh medicinal formulations
of Ayurveda, Unani and Siddha which are available in the public domain. To facilitate the protection

u
of country’s traditional knowledge, access of TKDL has been provided to International Patent offices

t
(IPOs) under International Agreement. In addition, pre-grant opposition on patent applications

.
along with prior art evidences from TKDL are submitted on a regular basis.

w w
65 Minamata Convention is deals with:

w
A. Methane
B. Mercury and its compounds
C. Arsenic Pollution
D. Wildlife Protection

Correct Answer : B

Answer Justification :

The Minamata Convention on Mercury is a global treaty to protect human health and the
environment from anthropogenic emissions and releases of mercury and mercury
compounds. It was adopted in 2013 in Kumamoto, Japan. It also controls the trans-boundary
movement of mercury.
It does not include natural emissions of mercury.

Hence, option (b) is correct.

41
Total Marks : 200
Online Prelims TEST - 20 (SUBJECT WISE)
( InsightsIAS Mock Test Series for UPSC Preliminary Exam 2020 ) Mark Scored : 0

66 Consider the following with respect to Integrated Development of Wildlife Habitats’ (IDWH)
1. It is a Central sector scheme for conservation of wildlife.
2. Under this, the financial assistance is provided to State/UT Governments for protection and
conservation of wildlife and its habitats in Protected Areas (PAs) only.

Which of the statements given above is/are correct?


A. 1 only
B. 2 only
C. Both 1 and 2
D. Neither 1 nor 2

Correct Answer : D

m
Answer Justification :

o
Integrated Development of Wildlife Habitats’ (IDWH) is an on-going Centrally Sponsored Scheme

. c
which has been made operational by adding more components and activities to the erstwhile

b
Centrally Sponsored Scheme - "Assistance for the Development of National Parks and Sanctuaries"

u
during the 11th Plan Period. Under IDWH, the financial assistance is provided to State/UT
Governments for protection and conservation of wildlife and its habitats in Protected Areas (PAs) as

h
well as outside PAs and also for the recovery programmes of the critically endangered

e
species.

u te
67 Consider the following with respect to Red Panda

t
.
1. It is listed as Critically Endangered under IUCN Red Book.
2. The geographical range is in Bhutan; China; India; Myanmar; Nepal.

w
3. Its habitat is montane forests with dense bamboo.

w
Which of the statements given above is/are correct?

w
A. 1 only
B. 1 and 2 only
C. 2 and 3 only
D. 1, 2 and 3

Correct Answer : C

Answer Justification :

Red Panda is listed as Endangered under IUCN Red Book

The current Red Panda distribution is detailed in three Population and Habitat Viability Analyses
(PHVAs) since 2010, covering all range states holding the species: Nepal (2010), China and
Myanmar (2012), and India and Bhutan (2013).

Red Panda is closely associated with montane forests (oak mixed; mixed broad-leaf conifer; and

42
Total Marks : 200
Online Prelims TEST - 20 (SUBJECT WISE)
( InsightsIAS Mock Test Series for UPSC Preliminary Exam 2020 ) Mark Scored : 0

conifer) with dense bamboo-thicket understorey.

68 Operation Save Kurma was initiated to protect

A. Turtles
B. Vultures
C. Gharial
D. Ganga Dolphins

Correct Answer : A

Answer Justification :

m
WCCB had convened a species-specific operation on turtles, code named OPERATION SAVE

o
KURMA. A total of 15, 739 live turtles were recovered from 45 suspects, having inter-state

c
linkages. The operation brought about an awareness among the enforcement agencies to focus on

.
the existing trade routes and major trade hubs in the country, which will be specifically focused in

b
future.

h u
69 Consider the following with reference to Wildlife Crime Control Bureau

e
1. It is a statutory multi-disciplinary body established under Prevention of Cruelty to Animals Acts

te
1960.

u
2. It is mandated to collect and collate intelligence related to organized wildlife crime activities and

t
establish centralized wildlife crime data bank.

w .
Which of the statements given above is/are correct?
A. 1 only

w
B. 2 only

w
C. Both 1 and 2
D. Neither 1 nor 2

Correct Answer : B

Answer Justification :

Wildlife Crime Control Bureau is a statutory multi-disciplinary body established by the


Government of India under the Ministry of Environment and Forests, to combat organized wildlife
crime in the country.

Under Section 38 (Z) of the Wild Life (Protection) Act, 1972, it is mandated

to collect and collate intelligence related to organized wildlife crime activities and to
disseminate the same to State and other enforcement agencies for immediate action so as to
apprehend the criminals;

43
Total Marks : 200
Online Prelims TEST - 20 (SUBJECT WISE)
( InsightsIAS Mock Test Series for UPSC Preliminary Exam 2020 ) Mark Scored : 0

to establish a centralized wildlife crime data bank;

co-ordinate actions by various agencies in connection with the enforcement of the provisions
of the Act;

assist foreign authorities and international organization concerned to facilitate co-ordination


and universal action for wildlife crime control;

capacity building of the wildlife crime enforcement agencies for scientific and professional
investigation into wildlife crimes and assist State Governments to ensure success in
prosecutions related to wildlife crimes;

m
and advise the Government of India on issues relating to wildlife crimes having national and

o
international ramifications, relevant policy and laws.

70 Important Bird and Biodiversity Areas (IBAs) is an initiative of

b . c
A. International Union for Conservation of Nature

h u
e
B. UNESCO

te
C. World Wide Fund for Nature
D. BirdLife International

Correct Answer : D

.tu
w
Answer Justification :

w
w
BirdLife’s Important Bird and Biodiversity Area (IBA) concept has been developed and applied
for over 30 years.

Initially, IBAs were identified only for terrestrial and freshwater environments, but over the past
decade, the IBA process and method has been adapted and applied in the marine realm.

IBAs are:

Places of international significance for the conservation of birds and other biodiversity

Recognised world-wide as practical tools for conservation

Distinct areas amenable to practical conservation action

Identified using robust, standardised criteria

44
Total Marks : 200
Online Prelims TEST - 20 (SUBJECT WISE)
( InsightsIAS Mock Test Series for UPSC Preliminary Exam 2020 ) Mark Scored : 0

Sites that together form part of a wider integrated approach to the conservation and
sustainable use of the natural environment

71 Which of the following countries is/are members of the International Centre for Integrated Mountain
Development?
1. Afghanistan
2. Bangladesh
3. China
4. Myanmar

Select the correct answer using the code given below:


A. 1 and 3 only
B. 1, 3 and 4 only
C. 2 and 4 only

m
D. 1, 2, 3 and 4

Correct Answer : D

. co
Answer Justification :

u b
e h
The International Centre for Integrated Mountain Development (ICIMOD) is a regional

te
intergovernmental learning and knowledge sharing centre serving the eight regional member
countries of the Hindu Kush Himalaya – Afghanistan, Bangladesh, Bhutan, China, India,

u
Myanmar, Nepal, and Pakistan – and based in Kathmandu, Nepal.

.t
http://www.icimod.org/?q=abt

w w
72 Which of the following areas is/are considered as priority areas by IUCN?
1. Biodiversity

w
2. Sustainable Energy
3. Green economy

Select the correct answer using the code given below:


A. 1 and 2 only
B. 2 and 3 only
C. 1 and 3 only
D. 1, 2 and 3

Correct Answer : D

Answer Justification :

All of the given areas are priority areas of IUCN.

IUCN was founded in October 1948 as the International Union for the Protection of Nature (or

45
Total Marks : 200
Online Prelims TEST - 20 (SUBJECT WISE)
( InsightsIAS Mock Test Series for UPSC Preliminary Exam 2020 ) Mark Scored : 0

IUPN) following an international conference in Fontainebleau, France.


The organization changed its name to the International Union for Conservation of Nature and
Natural Resources in 1956 with the acronym IUCN (or UICN) with its headquarters in Gland,
Switzerland

Priority Areas of IUCN


• Biodiversity
• Climate change
• Sustainable energy
• Human well-being
• Green economy

73 Which of the following countries is/are members of Global Tiger Forum (GTF)?
⌰〰〰

m
1. India

o
2. Indonesia

c
3. Cambodia

Select the correct answer using the code given below:

b .
u
A. 1 and 2 only

h
B. 2 and 3 only

e
C. 1 and 3 only

te
D. 1, 2 and 3

Correct Answer : C

.tu
w
Answer Justification :

w
GTF is the only inter-governmental body representing countries that still have wild tigers, and it is

w
responsible for facilitating, coordinating and strengthening these governments’ commitments and
actions towards saving tigers in the wild.

GTF was started in 1993 by tiger range states as a conduit for those countries to collaborate on a
global plan for tiger conservation. Seven tiger range countries (Bangladesh, Bhutan, India,
Cambodia, Laos, Nepal and Vietnam) are members of the GTF as well as the UK along with some
non-governmental organizations, including WWF and TRAFFIC.

Hence, option (c) is correct.

74 Consider the following statements


1. Jal Jeevan Mission aims to provide Functional Household Tap Connection (FHTC) to every rural
household by 2030.
2. Jal Shakti Ministry is the nodal ministry for this mission.

Which of the statements given above is/are correct?


A. 1 only

46
Total Marks : 200
Online Prelims TEST - 20 (SUBJECT WISE)
( InsightsIAS Mock Test Series for UPSC Preliminary Exam 2020 ) Mark Scored : 0

B. 2 only
C. Both 1 and 2
D. Neither 1 nor 2

Correct Answer : B

Answer Justification :

Jal Jeevan Mission, a central government initiative under the Ministry of Jal Shakti, aims to ensure
access of piped water for every household in India. It aims to provide Functional Household Tap
Connection (FHTC) to every rural household by 2024. Hence, statement 1 is incorrect.

Jal Shakti Ministry is the nodal ministry for this mission. Hence, statement 2 is correct.

75 The eBkray Platform has been launched by

o m
A.
B.
Ministry of Coal
Ministry of New and Renewable Energy

b . c
u
C. Ministry of Railways

h
D. Ministry of Finance

Correct Answer : D

te e
Answer Justification :

.tu
w
Recently Finance Ministery launched eBkray, an e-auction platform to enable online auction of
attached assets by banks. The eBkray platform provides navigational links to all PSB e-auction sites,

w
property search feature and presents single-window access to information on properties up for e-

w
auction, comparison of similar properties, and also contains videos and photographs of the uploaded
properties. Hence, option D is correct.

76 Consider the following statements regarding Forest Rights Act, 2006


1. The act recognize and vest the forest rights and occupation in forest land for dwelling Scheduled
Tribes only
2. The Gram Sabha is the authority to initiate the process for determining the nature and extent of
Individual Forest Rights
3. Land recognised under this act can be sold or transferred.

Which of the statements given above is/are correct?


A. 2 and 3 only
B. 2 only
C. 1 and 3 only
D. 1 and 2 only

47
Total Marks : 200
Online Prelims TEST - 20 (SUBJECT WISE)
( InsightsIAS Mock Test Series for UPSC Preliminary Exam 2020 ) Mark Scored : 0

Correct Answer : B

Answer Justification :

The act recognize and vest the forest rights and occupation for forest Dwelling Scheduled Tribes
(FDST) and Other Traditional Forest Dwellers (OTFD)who have been residing in such forests for
generations. Hence Statement 1 is incorrect.

The Gram Sabha is the authority to initiate the process for determining the nature and extent of
Individual Forest Rights (IFR) or Community Forest Rights (CFR) or both that may be given to FDST
and OTFD. Hence Statement 2 is correct.

Land recognised under this act cannot be sold or transferred. Hence Statement 3 is incorrect.

m
77 Consider the following statements regarding Banks Board Bureau

o
1. It is an autonomous body

c
2. It has genesis in the recommendations of the Committee to Review Governance of Boards of Banks

.
in India.

b
3. It is engaging with the Public Sector Banks (PSBs) to help build capacity to attract, retain and

u
nurture both talent and technology.

h
Which of the statements given above is/are correct?

e
A. 1 and 3 only

te
B. 1 and 2 only
C. 3 only

tu
D. 1, 2 and 3

Correct Answer : D

w .
w w
Answer Justification :

All the statements given above are correct.

The Banks Board Bureau has its genesis in the recommendations of The Committee to Review
Governance of Boards of Banks in India, May 2014. Thereafter, on February 28, 2016, the
Government of India, announced the constitution and composition of the Bureau. The Bureau
started functioning from April 01, 2016 as an autonomous recommendatory body.

As part of its mandate, and guided by a spirit of collaboration, the Bureau is engaging with various
stakeholders.

The objective of such engagement being to help prepare the banks in the public sector universe to
take on the competition, have the ability to appropriately manage and price risk across business
cycles, develop resilience to generate internal capital and have the capacity to generate external
capital warding of the moral hazard in counting on the scarce budgetary resources of tax payers.

The Bureau is also engaging with the Public Sector Banks (PSBs) to help build capacity to attract,

48
Total Marks : 200
Online Prelims TEST - 20 (SUBJECT WISE)
( InsightsIAS Mock Test Series for UPSC Preliminary Exam 2020 ) Mark Scored : 0

retain and nurture both talent and technology - the two key differentiators of business competencies
in the days to come. In its endeavour, the Bureau is mindful of the need to have a fully empowered
board in each and every PSB.

78 Consider the following statements


1. Galathea national park is located in Mizoram
2. Rohtang pass connects Kulu valley with Spiti valley of Himachal Pradesh
3. Lipulekh pass is located in Uttarkhand

Which of the statements given above is/are correct?


A. 1 and 2 only
B. 2 and 3 only
C. 3 only
D. 2 only

o m
. c
Correct Answer : B

b
Answer Justification :

h u
Galathea national park is located in Andaman and Nicobar Islands. Hence Statement 1 is

e
incorrect.

te
Rohtang pass connects Kulu valley with Spiti valley of Himachal Pradesh. Hence Statement 2 is

u
correct.

.t
Lipulekh is a Himalayan pass in Nepal near their trijunction with Nepal. Nepal has ongoing claims

w
to the southern side of the pass, which is controlled by India.

w
The pass is near the Chinese trading town of Taklakot (Purang) in Tibet and used since ancient
times by traders, mendicants and pilgrims transiting between India and Tibet. It is also used by

w
pilgrims to Kailas and Manasarovar.

49
Total Marks : 200
Online Prelims TEST - 20 (SUBJECT WISE)
( InsightsIAS Mock Test Series for UPSC Preliminary Exam 2020 ) Mark Scored : 0

79 Consider the following statements regarding Arctic Council


1. It was established by Ottawa Declaration
2. It has no programming budget
3. Chairmanship of the council rotates every two years among the eight national members

Which of the statements given above is/are correct?


A. 1 and 3 only
B. 2 and 3 only
C. 1 only
D. 1, 2 and 3

m
Correct Answer : D

Answer Justification :

. co
b
All the statements given above are correct.

h u
The Arctic Council is the leading intergovernmental forum promoting cooperation, coordination and

e
interaction among the Arctic States, Arctic indigenous communities and other Arctic inhabitants on

te
common Arctic issues, in particular on issues of sustainable development and environmental
protection in the Arctic. This article contains a backgrounder on the Arctic Council and its work.

tu
The Ottawa Declaration lists the following countries as Members of the Arctic Council: Canada, the

.
Kingdom of Denmark, Finland, Iceland, Norway, the Russian Federation, Sweden and the United

w
States.

w
The Chairmanship of the Arctic Council rotates every two years among the Arctic States

w
https://arctic-council.org/index.php/en/about-us

80 Consider the following statements regarding Jute production in India


1. Jute is a rabi crop
2. Almost 85% of world’s jute cultivation is concentrated in the Ganges Delta
3. Jute industries is predominantly independent from government support

Which of the statements given above is/are correct?


A. 1 only
B. 2 only
C. 2 and 3 only
D. 1 and 2 only

Correct Answer : B

50
Total Marks : 200
Online Prelims TEST - 20 (SUBJECT WISE)
( InsightsIAS Mock Test Series for UPSC Preliminary Exam 2020 ) Mark Scored : 0

Answer Justification :

Jute industry in India

Jute is an important Kharif crop. Hence Statement 1 is incorrect.

Jute is one of the important natural fibres after cotton in terms of cultivation and usage. Almost 85%
of world’s jute cultivation is concentrated in the Ganges Delta Hence Statement 2 is correct.

Jute industry is predominantly dependent on Government sector which purchases jute products
more than Rs. 5,500 crore every year. Hence Statement 3 is incorrect.

81 Consider the following statements regarding Atal Mission for Rejuvenation and Urban
Transformation (Amrut)

m
1. All Capital cities of states are not covered in AMRUT mission.

o
2. AMRUT will be implemented in 500 locations

Which of the statements given above is/are correct?


A. 1 only

b . c
u
B. 2 only

h
C. Both 1 and 2

e
D. Neither 1 nor 2

Correct Answer : C

tu te
.
Answer Justification :

w
w
Both the statements are correct.

w
AMRUT is the revamped version of the Jawaharlal Nehru National Urban Renewal Mission
(JNNURM). It was launched in June 2015 by Union Ministry of Housing and Urban Affairs.

Its objectives are:

1. Ensure that every household has access to a tap with the assured supply of water and a
sewerage connection.

2. Increase the amenity value of cities by developing greenery and well maintained open spaces
(e.g. parks) and

3. Reduce pollution by switching to public transport or constructing facilities for non-motorized


transport (e.g. walking and cycling).

4. AMRUT will be implemented in 500 locations.

51
Total Marks : 200
Online Prelims TEST - 20 (SUBJECT WISE)
( InsightsIAS Mock Test Series for UPSC Preliminary Exam 2020 ) Mark Scored : 0

Coverage under the scheme

All Cities and Towns with a population of over one lakh with notified Municipalities(including
Cantonment Boards)

All Capital Cities/Towns of States/ UTs not covered in above.

All Cities/ Towns classified as Heritage Cities by MoHUA under the HRIDAY Scheme.

Thirteen Cities and Towns on the stem of the main rivers with a population above 75,000 and less
than 1 lakh.

Ten Cities from hill states, islands and tourist destinations (not more than one from each State).

82 Consider the following statements regarding Equalisation Levy

m
1. It is an indirect tax

o
2. It is aimed at taxing business to business (B2B) transactions.

Which of the statements given above is/are correct?


A. 1 only

b . c
u
B. 2 only

h
C. Both 1 and 2

e
D. Neither 1 nor 2

Correct Answer : B

tu te
Answer Justification :

w .
w
Equalisation Levy is a direct tax. Hence Statement 1 is incorrect.

w
Equalisation Levy was introduced in India in 2016, with the intention of taxing the digital
transactions the income accruing to foreign e-commerce companies from India.

It is aimed at taxing business to business (B2B) transactions. Equalisation Levy is a direct tax,
which is withheld at the time of payment by the service recipient. Hence Statement 2 is correct.

83 Which of the following countries grant Generalized System of Preferences?

A. United States
B. Indonesia
C. China
D. India

Correct Answer : A

Answer Justification :
52
Total Marks : 200
Online Prelims TEST - 20 (SUBJECT WISE)
( InsightsIAS Mock Test Series for UPSC Preliminary Exam 2020 ) Mark Scored : 0

The Generalized System of Preferences (GSP), instituted in 1971 under the aegis of UNCTAD, has
contributed over the years to creating an enabling trading environment for developing countries.
The following 13 countries grant GSP preferences: Australia, Belarus, Canada, the European Union,
Iceland, Japan, Kazakhstan, New Zealand, Norway, the Russian Federation, Switzerland, Turkey
and the United States of America.

84 Which of the following tribes is/are present in Andaman and Nicobar Islands.
1. Onges
2. Sentineleses
3. Shompen

Select the correct answer using the code given below


A. 1, 2 and 3
B. 1 and 3 only

m
C. 1 and 2 only

o
D. 2 and 3 only

Correct Answer : A

b . c
Answer Justification :

h u
te
Imp tribes in Andaman and Nicobar Islands.

e
u
Great Andamanese -Strait Island is the part of North and Middle Andaman district which is the

t
home to Great Andamanese tribe, Fewer than 50 Great Andamanese are alive today.

w .
Jarawa - South Andaman and Middle Andaman Islands is inhabited by the Jarawa tribes, there are
only 300-400 people of this community alive today.

w w
Sentinelese -North Sentinel Island is part of North Andaman region which is home to the
Sentinelese tribe, only 50-100 tribes are alive today.

Onge - The Little Andaman Island is home to Ongetribes, these tribes are fewer than 100.

And Shompen.

85 Consider the following statements regarding Patent Cooperation Treaty (PCT)


1. PCT makes it possible to seek patent protection for an invention simultaneously in each of a large
number of countries
2. PCT is administered by World Trade Organisation (WTO).

Which of the statements given above is/are correct?


A. 1 only
B. 2 only
C. Both 1 and 2
D. Neither 1 nor 2

53
Total Marks : 200
Online Prelims TEST - 20 (SUBJECT WISE)
( InsightsIAS Mock Test Series for UPSC Preliminary Exm 2020 ) Mark Scored : 0

Correct Answer : A

Answer Justification :

The Patent Cooperation Treaty (PCT) makes it possible to seek patent protection for an invention
simultaneously in each of a large number of countries by filing an “international” patent application.

Such an application may be filed by anyone who is a national or resident of a PCT Contracting State.
It may generally be filed with the national patent office of the Contracting State of which the
applicant is a national or resident or, at the applicant’s option, with the International Bureau of
WIPO in Geneva.

PCT is administered by WIPO. Hence Statement 2 is incorrect.

m
86 Galo, Adi, Abor and Dufflas tribes, sometime seen in the news, are largely present in

A. Arunachal Pradesh

. co
b
B. Assam

u
C. Nagaland
D. Mizoram

e h
te
Correct Answer : A

tu
Answer Justification :

w .
https://www.thehindu.com/news/national/other-states/plan-to-have-specific-names-on-arunachal-st-li
st-cleared/article25896098.ece

w w
87 Which of the following organisation or agency releases House Price Index in India?

A. Reserve Bank of India


B. NITI Ayog
C. Dewan Housing Finance Corporation
D. Housing and Urban Development Corporation

Correct Answer : A

Answer Justification :

Reserve Bank released the quarterly house price index (HPI)1 (base: 2010-11=100) for Q1:2018-19,
based on transactions data received from housing registration authorities in ten major cities (viz.,
Mumbai, Delhi, Chennai, Kolkata, Bengaluru, Lucknow, Ahmedabad, Jaipur, Kanpur and Kochi).

54
Total Marks : 200
Online Prelims TEST - 20 (SUBJECT WISE)
( InsightsIAS Mock Test Series for UPSC Preliminary Exam 2020 ) Mark Scored : 0

https://www.rbi.org.in/Scripts/BS_PressReleaseDisplay.aspx?prid=45268

88 Consider the following statements regarding High-Temperature Proton Exchange Membrane


(HTPEM) technology developed in India
1. It is developed under the Public-Private Partnership (PPP) model by the Council of Scientific and
Industrial Research (CSIR) in partnership with Indian industries.
2. It takes nuclear fuel as the input and produces heat and water as its bi-products
3. It is suitable for distributed stationary power applications like small offices, commercial units etc

Which of the statements given above is/are correct?


A. 2 and 3 only
B. 1 and 3 only
C. 1 only
D. 1 and 2 only

o m
c
Correct Answer : B

Answer Justification :

b .
h u
India’s first indigenously developed high-temperature based Fuel Cell System was recently

e
introduced. It is a 5.0 kW fuel cell system that generates power in a green manner.

te
It takes methanol as the input and produces heat and water as its bi-products

tu
The developed fuel cells are based on High-Temperature Proton Exchange Membrane (HTPEM)

.
technology.

w w
Developed under the Public-Private Partnership (PPP) model by the Council of Scientific
and Industrial Research (CSIR) in partnership with Indian industries.

wBuilt under India’s flagship programme named ‘New Millennium Indian Technology
Leadership Initiative (NMITLI)’.

Applications:

1. Suitable for distributed stationary power applications like; for small offices, commercial
units, data centers etc.; where highly reliable power is essential with simultaneous
requirement for air-conditioning.

2. It will also meet the requirement of efficient, clean and reliable backup power generator for
telecom towers, remote locations and strategic applications as well.

55
Total Marks : 200
Online Prelims TEST - 20 (SUBJECT WISE)
( InsightsIAS Mock Test Series for UPSC Preliminary Exam 2020 ) Mark Scored : 0

3. Replace Diesel Generating (DG) sets and help reduce India’s dependence on crude oil.

https://www.insightsonindia.com/2019/09/27/high-temperature-proton-exchange-membrane-htpem-t
echnology/

89 Consider the following statements regarding Global Internet Forum to Counter Terrorism (GIFCT)
1. It was formally established in July 2017 as a group of companies, dedicated to disrupting terrorist
abuse of members’ digital platforms.
2. The original Forum was led by a rotating chair drawn from the founding four companies namely
Facebook, Microsoft, Twitter and YouTube

Which of the statements given above is/are correct?


A. 1 only

m
B. 2 only

o
C. Both 1 and 2

c
D. Neither 1 nor 2

b .
u
Correct Answer : C

Answer Justification :

e h
te
Both the statements are correct.

tu
The Global Internet Forum to Counter Terrorism (GIFCT) was formally established in July 2017 as a

.
group of companies, dedicated to disrupting terrorist abuse of members’ digital platforms. The
original Forum was led by a rotating chair drawn from the founding four companies—Facebook,

w
Microsoft, Twitter and YouTube—and managed a program of knowledge-sharing, technical

w
collaboration and shared research.

w
https://www.gifct.org/about/

90 Microhyla eos, the new discovered frog species is found in

A. Madhya Pradesh
B. Karnataka
C. Andaman and Nicobar
D. Arunachal Pradesh

Correct Answer : D

Answer Justification :

It is found in Arunachal Pradesh

56
Total Marks : 200
Online Prelims TEST - 20 (SUBJECT WISE)
( InsightsIAS Mock Test Series for UPSC Preliminary Exam 2020 ) Mark Scored : 0

https://www.thehindu.com/news/national/arunachal-yields-a-new-frog-species/article29494090.ece

91 Consider the following statements regarding Shondol dance


1. It is native to Sikkim
2. It is associated with Buddhist religion and culture

Which of the statements given above is/are correct?


A. 1 only
B. 2 only
C. Both 1 and 2
D. Neither 1 nor 2

Correct Answer : B

Answer Justification :

o m
. c
Ladakhi Shondol dance has created history yet again by breaking into the Guinness book of world
records as the largest Ladakhi dance. Hence Statement 1 is incorrect

b
h u
Around 408 women artists in traditional dresses performed gracefully the Shondol dance on the

e
occasion of the Annual Naropa Festival.

te
The ongoing Buddhist carnival near the 11th-century world-famous Hemis Monastery in Ladakh has

u
ended last evening with creating history as one of the folk dance namely Shondol has broke the

t
previous record of 299 artists dance of Shondol in the 2018 festival.

w .
http://www.newsonair.com/News?title=Ladakhi-Shondol-dance-created-history&id=371868

w
92 Consider the following statements regarding Higher Education Finance Agency (HEFA),

w
1. HEFA is a joint venture of Ministry of HRD, GOI and Canara Bank with an agreed equity
participation in the ratio of 60% and 40% respectively.
2. HEFA is registered under Section 8 under the Companies Act, 2013 as a Union Government
company and as Non–deposit taking NBFC with RBI.

Which of the statements given above is/are correct?


A. 1 only
B. 2 only
C. Both 1 and 2
D. Neither 1 nor 2

Correct Answer : B

Answer Justification :

HEFA incorporated on 31st May 2017, is a joint venture of Ministry of HRD, GOI and Canara Bank

57
Total Marks : 200
Online Prelims TEST - 20 (SUBJECT WISE)
( InsightsIAS Mock Test Series for UPSC Preliminary Exam 2020 ) Mark Scored : 0

with an agreed equity participation in the ratio of 91% and 9% respectively. Hence Statement 1 is
incorrect.

Higher Education Financing Agency (HEFA) is a joint venture of MHRD Government of India and
Canara Bank for financing creation of capital assets in premier educational institutions in India as
part of rising 2022 HEFA’s scope is greatly expanded to cover school education, educational
institutes under Ministry of health etc.

HEFA is registered under Section 8 [Not-for-profit] under the Companies Act 2013 as a Union Govt
company and as Non–deposit taking NBFC (NBFC-ND-Type II) with RBI. Hence Statement 2 is
correct

93 Which of the following countries borders Myanmar?

m
A. Bangladesh, Thailand, India and Vietnam

o
B. Cambodia, Laos and Malaysia and Bangladesh

c
C. Thailand, Vietnam and Malaysia

.
D. Thailand, Laos, Bangladesh and China

u b
h
Correct Answer : D

Answer Justification :

te e
.tu
w w
w
94 GAFA tax recently seen in news is related to

A. Tax on new start-ups with annual turn-over of more than 1 crore


B. Tax on Renewable energy products
C. Tax on large technology and internet companies
D. None of the above

Correct Answer : C

Answer Justification :

58
Total Marks : 200
Online Prelims TEST - 20 (SUBJECT WISE)
( InsightsIAS Mock Test Series for UPSC Preliminary Exam 2020 ) Mark Scored : 0

French finance minister Bruno Le Maire announced the introduction of a GAFA tax—named after
Google, Apple, Facebook, Amazon—on large technology and internet companies in France from 1
January 2019. The French proposal is set to target only those profitable companies that have heavy
annual global revenue.

95 Consider the following statements regarding African swine fever


1. It is a severe viral disease affecting domestic and wild pigs
2. It is threat to human beings since it can spread from animals.
3. It is restricted to Africa only

Which of the statements given above is/are correct?


A. 1 only
B. 2 only
C. 1 and 2 only

m
D. 1, 2 and 3

Correct Answer : A

. co
Answer Justification :

u b
e h
African swine fever (ASF) is a severe viral disease affecting domestic and wild pigs. Hence

te
Statement 1 is correct.

u
It is responsible for serious production and economic losses; This transboundary animal disease

t
(TAD) can be spread by live or dead pigs, domestic or wild, and pork products;

w .
Furthermore, transmission can also occur via contaminated feed and fomites (non-living objects)
such as shoes, clothes, vehicles, knives, equipment etc., due to the high environmental resistance of

w
ASF virus

w
ASF is not a threat to human beings since it only spreads from animals to other animals. Hence
Statement 2 is incorrect.

ASF has been seen in Asian countries like China, Philippines recently. Hence Statement 3 is
incorrect

96 Consider the following statements regarding Participatory Guarantee Scheme (PGS)


1. It is an internationally applicable organic quality assurance system implemented and controlled by
the committed organic farmer-producers through active participation.
2. PGS Organic India Council was set up by Ministry of Commerce and Industry

Which of the statements given above is/are correct?


A. 1 only
B. 2 only
C. Both 1 and 2
D. Neither 1 nor 2

59
Total Marks : 200
Online Prelims TEST - 20 (SUBJECT WISE)
( InsightsIAS Mock Test Series for UPSC Preliminary Exam 2020 ) Mark Scored : 0

Correct Answer : A

Answer Justification :

The PGS is an internationally applicable organic quality assurance system [like ISO 9000]
implemented and controlled by the committed organic farmer-producers through active
participation, along with the consumers, in the process based on verifiable trust. It is not an
“inspection raj” certification system but, rather, one that is based on personal integrity and peer
pressure. Hence Statement 1 is correct.

The PGS Organic India Council was set up after a consultation process in 2006. It functioned as
an informal coalition of Voluntary Organizations or NGOs committed to the promotion of
organic food production for domestic consumption in India, with export not being a priority at
all. In April 2011, it was formally registered as a society in Goa as Participatory Guarantee Systems

m
Organic Council (PGSOC). Hence Statement 2 is incorrect.

http://pgsorganic.in/

. co
u b
97 Project REPLAN (Reducing Plastic in Nature), sometime seen in the news, is launched by

h
A. Ministry of Environment, Forest and Climate Change

e
B. Khadi and Village Industries Commission

te
C. Central Pollution Control Board

u
D. None of the above

.t
w
Correct Answer : B

w
Answer Justification :

w
It is launched by Khadi and Village Industries Commission (KVIC). It aims to make carry bags
by mixing processed and treated plastic waste with cotton fibre rags in the ratio 20:80.

98 Consider the following statements regarding ‘Hope Spots’.


1. Andaman and Nicobar Islands and Lakshadweep islands are the first in India to make it to the list of
global hope spots.
2. A hope spot is an area of an ocean that needs special protection because of its wildlife and
significant underwater habitats.

Which of the statements given above is/are correct?


A. 1 only
B. 2 only
C. Both 1 and 2
D. Neither 1 nor 2

60
Total Marks : 200
Online Prelims TEST - 20 (SUBJECT WISE)
( InsightsIAS Mock Test Series for UPSC Preliminary Exam 2020 ) Mark Scored : 0

Correct Answer : C

Answer Justification :

Both the statements are correct.

Andaman and Nicobar Islands and Lakshadweep islands have recently been named as the new
“hope spots” by the International Union for Conservation of Nature (IUCN) and Mission Blue, an
organization involved in the study of oceans.

A hope spot is an area of an ocean that needs special protection because of its wildlife and
significant underwater habitats.

https://mission-blue.org/hope-spots/

m
99 Consider the following statements

o
1. Western Dedicated Freight Corridor passes through 5 states

. c
2. Eastern Dedicated Freight Corridor passes through 6 states

Which of the statements given above is/are correct?


A. 1 only

u b
h
B. 2 only

e
C. Both 1 and 2

te
D. Neither 1 nor 2

Correct Answer : C

.tu
w
Answer Justification :

w w
Both the statements are correct.

Eastern Dedicated Freight Corridor (EDFC) is from Ludhiana to Dankuni (1318 Kms Ludhiana
to Sonnagar and 538 Kms Sonnagar to Dankuni) and Western Dedicated Freight Corridor (WDFC) is
from Jawaharlal Nehru Port Terminal (JNPT) to Dadri (1504 Kms).

Western DFC passes through States of Uttar Pradesh, Haryana, Rajasthan, Gujarat and
Maharashtra and Eastern DFC through States of Punjab, Haryana, Uttar Pradesh, Bihar, Jharkhand
and West Bengal.

https://www.insightsonindia.com/2019/12/28/western-dedicated-freight-corridor/

100 Consider the following statements regarding SnowEx


1. It is a five year program initiated and funded by European Space Agency
2. It aims to address the most important gaps in snow remote sensing knowledge
3. The geographical focus of project is Northern hemisphere

Which of the statements given above is/are correct?

61
Total Marks : 200
Online Prelims TEST - 20 (SUBJECT WISE)
( InsightsIAS Mock Test Series for UPSC Preliminary Exam 2020 ) Mark Scored : 0

A. 1 and 2 only
B. 2 only
C. 1 and 3 only
D. 2 and 3 only

Correct Answer : D

Answer Justification :

SnowEx is a five year program initiated and funded by NASA THP to address the most
important gaps in snow remote sensing knowledge.

It focuses on airborne campaigns and field work, and on comparing the various sensing
technologies, from the mature to the more experimental, in globally-representative types of snow.

o m
The geographical focus of SnowEx is proposed as North America which contains the six broad

c
snow climate categories identified in the literature: tundra (alpine or Arctic), taiga (Boreal forest),

.
warm (temperate) forest, maritime, prairie, and ephemeral.

https://snow.nasa.gov/campaigns/snowex

u b
e h
tu te
w .
w w

62

You might also like